Urgenthomework logo
UrgentHomeWork
Live chat

Loading..

ATI Pharmacology Proctored Test

  

  1. 1) A nurse is caring for a client with hyperparathyroidism and notes that the client's serum calcium level is 13 mg/dL. Which medication should the nurse prepare to administer as prescribed to the client?
  2. Calcium chloride
  3. Calcium gluconate
  4. Calcitonin (Miacalcin)
  5. Large doses of vitamin D
  6. 2.) Oral iron supplements are prescribed for a 6-year-old child with iron deficiency anemia. The nurse instructs the mother to administer the iron with which best food item? 1. Milk
  7. Water
  8. Apple juice
  9. Orange juice
  10. 3.) Salicylic acid is prescribed for a client with a diagnosis of psoriasis. The nurse monitors the client, knowing that which of the following would indicate the presence of systemic toxicity from this medication?
  11. Tinnitus
  12. Diarrhea
  13. Constipation
  14. Decreased respirations
  15. 4.) The camp nurse asks the children preparing to swim in the lake if they have applied sunscreen. The nurse reminds the children that chemical sunscreens are most effective when applied: 1. Immediately before swimming
  16. 15 minutes before exposure to the sun
  17. Immediately before exposure to the sun At least 30 minutes before exposure to the sun
  18. 5.) Mafenide acetate (Sulfamylon) is prescribed for the client with a burn injury. When applying the medication, the client complains of local discomfort and burning. Which of the following is the most appropriate nursing action? 1. Notifying the registered nurse
  19. Discontinuing the medication
  20. Informing the client that this is normal Applying a thinner film than prescribed to the burn site
  1. Calcitonin (Miacalcin) Rationale:

The normal serum calcium level is 8.6 to 10.0 mg/dL. This client is experiencing hypercalcemia. Calcium gluconate and calcium chloride are medications used for the treatment of tetany, which occurs as a result of acute hypocalcemia. In hypercalcemia, large doses of vitamin D need to be avoided. Calcitonin, a thyroid hormone, decreases the plasma calcium level by inhibiting bone resorption and lowering the serum calcium concentration.

  1. Orange juice Rationale:

Vitamin C increases the absorption of iron by the body. The mother should be instructed to administer the medication with a citrus fruit or a juice that is high in vitamin C. Milk may affect absorption of the iron. Water will not assist in absorption. Orange juice contains a greater amount of vitamin C than apple juice.

  1. Tinnitus

Rationale:

Salicylic acid is absorbed readily through the skin, and systemic toxicity (salicylism) can result. Symptoms include tinnitus, dizziness, hyperpnea, and psychological disturbances. Constipation and diarrhea are not associated with salicylism.  

  1. At least 30 minutes before exposure to the sun Rationale:

Sunscreens are most effective when applied at least 30 minutes before exposure to the sun so that they can penetrate the skin. All sunscreens should be reapplied after swimming or sweating.

  1. Informing the client that this is normal Rationale:

Mafenide acetate is bacteriostatic for gram-negative and gram-positive organisms and is used to treat burns to reduce bacteria present in avascular tissues. The client should be informed that the medication will cause local discomfort and burning and that this is a normal reaction; therefore options 1, 2, and 4 are incorrect

  1. 6. ) The burn client is receiving treatments of 1. Hyperventilation

Rationale: topical mafenide acetate (Sulfamylon) to the

Mafenide acetate is a carbonic anhydrase inhibitor and can suppressrenal excretion site of injury. The nurse monitors the client, of acid, thereby causing acidosis. Clients receiving this treatment should be knowing that which of the following indicates monitored for signs of an acid-base imbalance (hyperventilation). If this occurs, the that a systemic effect has occurred? medication should be discontinued for 1 to 2 days. Options 3 and 4 describe local

1.Hyperventilation rather than systemic effects. An elevated blood pressure may be expected from the

2.Elevated blood pressure pain that occurs with a burn injury.

3.Local pain at the burn site

        4.Local rash at the burn site                                   

7. 7.) Isotretinoin is prescribed for a client with severe acne. Before the administration of this medication, the nurse anticipates that which laboratory test will be prescribed?

1.  Platelet count

2.  Triglyceride level

3.  Complete blood count                

4.  White blood cell count                

2.Triglyceride level Rationale:

Isotretinoin can elevate triglyceride levels. Blood triglyceride levels should be measured before treatment and periodically thereafter until the effect on the triglycerides has been evaluated. Options 1, 3, and 4 do not need to be monitored specifically during this treatment.

8. 8.) A client with severe acne is seen in the clinic and the health care provider (HCP) prescribes isotretinoin. The nurse reviews the client's medication record and would contact the (HCP) if the client is taking which medication?

1.  Vitamin A          

2.  Digoxin (Lanoxin)             

3.  Furosemide (Lasix)             

4.  Phenytoin (Dilantin)             

1.Vitamin A Rationale:

Isotretinoin is a metabolite of vitamin A and can produce generalized intensification of isotretinoin toxicity. Because of the potential for increased toxicity, vitamin A supplements should be discontinued before isotretinoin therapy. Options 2, 3, and 4 are not contraindicated with the use of isotretinoin.

9. 9.) The nurse is applying a topical corticosteroid to a client with eczema. The nurse would monitor for the potential for increased systemic absorption of the medication if the medication were being applied to which of the following body areas?

1.  Back  

2.  Axilla  

3.  Soles of the feet     

4.  Palms of the hands             

2. Axilla

Rationale:

Topical corticosteroids can be absorbed into the systemic circulation. Absorption is higher from regions where the skin is especially permeable (scalp, axilla, face, eyelids, neck, perineum, genitalia), and lower from regions in which permeability is poor (back, palms, soles).

10. 10.) The clinic nurse is performing an admission assessment on a client. The nurse notes that the client is taking azelaic acid (Azelex). Because of the medication prescription, the nurse would suspect that the client is being treated for:

1.  Acne

2.  Eczema

3.  Hair loss

4.  Herpes simplex

1. Acne

Rationale:

Azelaic acid is a topical medication used to treat mild to moderate acne. The acid appears to work by suppressing the growth of Propionibacterium acnes and decreasing the proliferation of keratinocytes. Options 2, 3, and 4 are incorrect.

  1. "Themedication will permanently stain my skin."
  2. 11.) The health care provider has prescribed silver

Rationale: sulfadiazine (Silvadene) for the client with a partial-

Silver sulfadiazine (Silvadene) is an antibacterial that has a broad spectrum thickness burn, which has cultured positive for gram- of activity against gram-negative bacteria, gram-positive bacteria, and negative bacteria. The nurse is reinforcing yeast. It is applied directly to the wound to assist in healing. It does not information to the client about the medication. Which

stain the skin. statement made by the client indicates a lack of understanding about the treatments?  

  1. "The medication is an antibacterial."  
  2. "The medication will help heal the burn."  
  3. "The medication will permanently stain my skin." "The medication should be applied directly to the  wound."         
  4. Notify the registered nurse.
  5. 12.) A nurse is caring for a client who is receiving an

Rationale: intravenous (IV) infusion of an antineoplastic

When antineoplastic medications (Chemotheraputic Agents) are medication. During the infusion, the client complains administered via IV, great care must be taken to prevent the medication from of pain at the insertion site. During an inspection of escaping into the tissues surrounding the injection site, because pain, tissue the site, the nurse notes redness and swelling and damage, and necrosis can result. The nurse monitors for signs of that the rate of infusion of the medication has extravasation, such as redness or swelling at the insertion site and a slowed. The nurse should take which appropriate decreased infusion rate. If extravasation occurs, the registered nurse needs action?

  1. Notify the registered nurse. to be notified; he or she will then contact the health care provider.
  2. Administer pain medication to reduce the                 
  3. Apply ice and maintain the infusion rate, as                
  4. Elevate the extremity of the IV site, and slow the  
  5. Pulmonary function studies
  6. 13.) The client with squamous cell carcinoma of the

Rationale: larynx is receiving bleomycin intravenously. The

Bleomycin is an antineoplastic medication (Chemotheraputic Agents) that nurse caring for the client anticipates that which can cause interstitial pneumonitis, which can progress to pulmonary fibrosis. diagnostic study will be prescribed?

  1. Echocardiography Pulmonary function studies along with hematological, hepatic, and renal
  2. Electrocardiography function tests need to be monitored. The nurse needs to monitor lung
  3. Cervical radiography           sounds for dyspnea and crackles, which indicate pulmonary toxicity. The
  4. Pulmonary function studies medication needs to be discontinued immediately if pulmonary toxicity

            occurs. Options 1, 2, and 3 are unrelated to the specific use of this medication.  

14. 14.) The client with acute myelocytic leukemia is being treated with busulfan (Myleran). Which laboratory value would the nurse specifically monitor during treatment with this medication?

1.  Clotting time

2.  Uric acid level

3.  Potassium level

4.  Blood glucose level

2.Uric acid level Rationale:

Busulfan (Myleran) can cause an increase in the uric acid level. Hyperuricemia can produce uric acid nephropathy, renal stones, and acute renal failure. Options 1, 3, and 4 are not specifically related to this medication.

  1. 15.) The client with small cell lung cancer is being treated with etoposide (VePesid). The nurse who is assisting in caring for the client during its administration understands that which side effect is specifically associated with this medication? 1. Alopecia
  2. Chest pain
  3. Pulmonary fibrosis
  4. Orthostatic hypotension
  5. 16.) The clinic nurse is reviewing a teaching plan for the client receiving an antineoplastic medication. When implementing the plan, the nurse tells the client:
  6. To take aspirin (acetylsalicylic acid) as needed for headache
  7. Drink beverages containing alcohol in moderate amounts each evening
  8. Consult with health care providers (HCPs) before receiving immunizations That it is not necessary to consult HCPs before receiving a flu vaccine at the local health fair
  9. 17.) The client with ovarian cancer is being treated with vincristine (Oncovin). The nurse monitors the client, knowing that which of the following indicates a side effect specific to this medication? 1. Diarrhea
  10. Hair loss
  11. Chest pain
  12. Numbness and tingling in the fingers and toes
  13. 18.) The nurse is reviewing the history and physical examination of a client who will be receiving asparaginase (Elspar), an antineoplastic agent. The nurse consults with the registered nurse regarding the administration of the medication if which of the following is documented in the client's history?
  14. Pancreatitis
  15. Diabetes mellitus
  16. Myocardial infarction
  17. Chronic obstructive pulmonary disease
  1. Orthostatic hypotension Rationale:

A side effect specific to etoposide is orthostatic hypotension. Theclient's blood pressure is monitored during the infusion. Hair loss occurs with nearly all the antineoplastic medications. Chest pain and pulmonary fibrosis are unrelated to this medication.


  

  1. Consult with health care providers (HCPs) before receiving immunizations Rationale:

Because antineoplastic medications lower the resistance of the body, clients must be informed not to receive immunizations without a HCP's approval. Clients also need to avoid contact with individuals who have recently received a live virus vaccine. Clients need to avoid aspirin and aspirin-containing products to minimize the risk of bleeding, and they need to avoid alcohol to minimize the risk of toxicity and side effects.

  1. Numbness and tingling in the fingers and toes Rationale:

A side effect specific to vincristine is peripheral neuropathy, which occurs in almost every client. Peripheral neuropathy can be manifested as numbness and tingling in the fingers and toes. Depression of the Achilles tendon reflex may be the first clinical sign indicating peripheral neuropathy. Constipation rather than diarrhea is most likely to occur with this medication, although diarrhea may occur occasionally. Hair loss occurs with nearly all the antineoplastic medications.Chest pain is unrelated to this medication.

1.Pancreatitis Rationale:

Asparaginase (Elspar) is contraindicated if hypersensitivity exists, in pancreatitis, or if the client has a history of pancreatitis. The medication impairs pancreatic function and pancreatic function tests should be performed before therapy begins and when a week or more has elapsed between administration of the doses. The client needs to be monitored for signs of pancreatitis, which include nausea, vomiting, and abdominal pain. The conditions noted in options 2, 3, and 4 are not contraindicated with this medication.

  1. 19.) Tamoxifen is prescribed for the client with metastatic breast carcinoma. The nurse understands that the primary action of this medication is to:
  2. Increase DNA and RNA synthesis.
  3. Promote the biosynthesis of nucleic acids.
  4. Increase estrogen concentration and estrogen response.
  5. Compete with estradiol for binding to estrogen in tissues containing high concentrations of receptors.
  6. Compete with estradiolfor binding to estrogen in tissues containing high concentrations of receptors.

Rationale:

Tamoxifen is an antineoplastic medication that competes with estradiol for binding to estrogen in tissues containing high concentrations of receptors. Tamoxifen is used to treat metastatic breast carcinoma in women and men. Tamoxifen is also effective in delaying the recurrence of cancer following mastectomy. Tamoxifen reduces DNA synthesis and estrogen response.

20. 20.) The client with metastatic breast cancer is receiving tamoxifen. The nurse specifically monitors which laboratory value while the client is taking this medication?

1.  Glucose level

2.  Calcium level

3.  Potassium level

4.  Prothrombin time       

2. Calcium level Rationale:

Tamoxifen may increase calcium, cholesterol, and triglyceride levels. Before the initiation of therapy, a complete blood count, platelet count, and serum calcium levels should be assessed. These blood levels, along with cholesterol and triglyceride levels, should be monitored periodically during therapy. The nurse should assess for hypercalcemia while the client is taking this medication. Signs of hypercalcemia include increased urine volume, excessive thirst, nausea, vomiting, constipation, hypotonicity of muscles, and deep bone and flank pain.

21. 21.) A nurse is assisting with caring for a client with cancer who is receiving cisplatin. Select the adverse effects that the nurse monitors for that are associated with this medication. Select all that apply. 1. Tinnitus

2.  Ototoxicity

3.  Hyperkalemia

4.  Hypercalcemia          

5.  Nephrotoxicity           

6.  Hypomagnesemia     

1. Tinnitus

2. Ototoxicity

5.Nephrotoxicity6.Hypomagnesemia Rationale:

Cisplatin is an alkylating medication. Alkylating medications are cell cycle phase-nonspecific medications that affect the synthesis of DNA by causing the cross-linking of DNA to inhibit cell reproduction. Cisplatin may cause ototoxicity, tinnitus, hypokalemia, hypocalcemia, hypomagnesemia, and nephrotoxicity. Amifostine (Ethyol) may be administered before cisplatin to reduce the potential for renal toxicity.

22. 22.) A nurse is caring for a client after thyroidectomy and notes that calcium gluconate is prescribed for the client. The nurse determines that this medication has been prescribed to:

1.  Treat thyroid storm.

2.  Prevent cardiac irritability.

3.  Treat hypocalcemic tetany. 4. Stimulate the release of parathyroid hormone.

3. Treat hypocalcemic tetany.

Rationale:

Hypocalcemia can develop after thyroidectomy if the parathyroid glands are accidentally removed or injured during surgery. Manifestations develop 1 to 7 days after surgery. If the client develops numbness and tingling around the mouth, fingertips, or toes or muscle spasms or twitching, the health care provider is notified immediately. Calcium gluconate should be kept at the bedside.

  1. ) A client who has been newly diagnosed with diabetes mellitus has been stabilized with daily insulin injections. Which information should the nurse teach when carrying out plans for discharge?
    1. Keep insulin vials refrigerated at all times.
    2. Rotate the insulin injection sites systematically.
    3. Increase the amount of insulin before unusual exercise.
    4. Monitor the urine acetone level to determine the insulin dosage.
  2. ) A nurse is reinforcing teaching for a client regarding how to mix regular insulin and NPH insulin in the same syringe. Which of the following actions, if performed by the client, indicates the need for further teaching? 1. Withdraws the NPH insulin first
    1. Withdraws the regular insulin first
    2. Injects air into NPH insulin vial

first

  • Injects an amount of air equal to the desired dose of insulin into the vial
  1. ) A home care nurse visits a client recently diagnosed with diabetes mellitus who is taking Humulin NPH insulin daily. The client asks the nurse how to store the unopened vials of insulin. The nurse tells the client to:
    1. Freeze the insulin.
    2. Refrigerate the insulin.
    3. Store the insulin in a dark, dry place.
    4. Keep the insulin at room temperature.
    5. Rotate the insulin injection sites systematically.

Rationale:

Insulin dosages should not be adjusted or increased before unusual exercise.If acetone is found in the urine, it may possibly indicate the need for additional insulin. To minimize the discomfort associated with insulin injections, the insulin should be administered at room temperature. Injection sites should be systematically rotated from one area to another. The client should be instructed to give injections in one area, about 1 inch apart, until the whole area has been used and then to change to another site. This prevents dramatic changes in daily insulin absorption.

  1. Withdraws the NPH insulin first Rationale:

When preparing a mixture of regular insulin with another insulin preparation, the regular insulin is drawn into the syringe first. This sequence will avoid contaminating the vial of regular insulin with insulin of another type. Options 2, 3, and 4 identify the correct actions for preparing NPH and regular insulin.

  1. Refrigerate the insulin.

Rationale:

Insulin in unopened vials should be stored under refrigeration until needed. Vials should not be frozen. When stored unopened under refrigeration, insulin can be used up to the expiration date on the vial. Options 1, 3, and 4 are incorrect.

26. 26.) Glimepiride (Amaryl) is prescribed for a client with diabetes mellitus. A nurse reinforces instructions for the client and tells the client to avoid which of the following while taking this medication? 1. Alcohol

2.  Organ meats

3.  Whole-grain cereals

4.  Carbonated beverages

1. Alcohol

Rationale:

When alcohol is combined with glimepiride (Amaryl), a disulfiram-like reaction may occur. This syndrome includes flushing, palpitations, and nausea. Alcohol can also potentiate the hypoglycemic effects of the medication. Clients need to be instructed to avoid alcohol consumption while taking this medication. The items in options 2, 3, and 4 do not need to be avoided.

  1. Useof nitroglycerin
  2. ) Sildenafil (Viagra) is prescribed to treat a client

Rationale: with erectile dysfunction. A nurse reviews the

Sildenafil (Viagra) enhances the vasodilating effect of nitric oxidein the client's medical record and would question the corpus cavernosum of the penis, thus sustaining an erection. Because of the prescription if which of the following is noted in the effect of the medication, it is contraindicated with concurrent use of organic client's history?

  1. Neuralgia nitrates and nitroglycerin. Sildenafil is not contraindicated with the use of
  2. Insomnia Neuralgia and insomnia are side effects of the medication.
  3. Use of nitroglycerin  
  4. Use of multivitamins  
    1. The medication is withheld and the HCP is called to question the
  5. ) The health care provider (HCP) prescribes

prescription for the client. exenatide (Byetta) for a client with type [1][2] diabetes

Rationale: mellitus who takes insulin. The nurse knows that

Exenatide (Byetta) is an incretin mimetic used for type 2 diabetes mellitus which of the following is the appropriate only. It is not recommended for clients taking insulin. Hence, the nurse intervention?

  1. The medication is administered within 60 minutes should hold the medication and question the HCP regarding this prescription. Although options [3] and 3 are correct statements about the before the morning and evening meal.
  2. The medication is withheld and the HCP is called medication, in this situation the medication should not be administered. The medication is packaged in prefilled pens ready for injection without the to question the prescription for the client.
  3. The client is monitored for gastrointestinal side need for drawing it up into another syringe.

effects after administration of the medication.  4. The insulin is withdrawn from the Penlet into an  insulin syringe to prepare for administration.  

  1. 4 to 12 hours after administration
  1. ) A client is taking Humulin NPH insulin daily

Rationale: every morning. The nurse reinforces instructions for

Humulin NPH is an intermediate-acting insulin. The onset of action is 1.5 the client and tells the client that the most likely hours, it peaks in 4 to 12 hours, and its duration of action is 24 hours. time for a hypoglycemic reaction to occur is:

  1. [4][5] to [6] hours after administration Hypoglycemic reactions most likely occur during peak time.
  2. 4 to 12 hours after administration  
  3. 16 to 18 hours after administration  
  4. 18 to 24 hours after administration  

31[7]. 31.) A community health nurse visits a client at home. Prednisone 10 mg orally daily has been prescribed for the client and the nurse reinforces teaching for the client about the medication. Which statement, if made by the client, indicates that further teaching is necessary?

  1. "I can take aspirin or my antihistamine if I need it."
  2. "I need to take the medication every day at the same time."
  3. "I need to avoid coffee, tea, cola, and chocolate in my diet."

  • "If I gain more than 5 pounds a week, I will call my doctor."
  1. ) Desmopressin acetate (DDAVP) is prescribed for the treatment of diabetes insipidus. The nurse monitors the client after medication administration for which therapeutic response? 1. Decreased urinary output
    1. Decreased blood pressure
    2. Decreased peripheral edema
    3. Decreased blood glucose level
  2. ) The home health care nurse is visiting a client who was recently diagnosed with type [8] diabetes mellitus. The client is prescribed repaglinide (Prandin) and metformin (Glucophage) and asks the nurse to explain these medications. The nurse should reinforce which instructions to the client? Select all that apply.
    1. Diarrhea can occur secondary to the metformin.
    2. The repaglinide is not taken if a meal is skipped.
    3. The repaglinide is taken 30 minutes before eating.
    4. Candy or another simple sugar is carried and used to treat mild hypoglycemia episodes.
    5. Metformin increases hepatic glucose production to prevent hypoglycemia associated with repaglinide.
    6. Muscle pain is an expected side effect of metformin and may be treated with acetaminophen (Tylenol).
  1. "I can take aspirinor my antihistamine if I needit." Rationale:

Aspirin and other over-the-counter medications should not be taken unless theclient consults with the health care provider (HCP). The client needs to take the medication at the same time every day and should be instructed not to stop the medication. A slight weight gain as a result of an improved appetite is expected, but after the dosage is stabilized, a weight gain of 5 lb or more weekly should be reported to the HCP.Caffeine-containing foods and fluids need to be avoided because they may contribute to steroid-ulcer development.

  1. Decreased urinary output Rationale:

Desmopressin promotes renal conservation of water. The hormone carries out this action by acting on the collecting ducts of the kidney to increase their permeability to water, which results in increased water reabsorption. The therapeutic effect of this medication would be manifested by a decreased urine output. Options 2, [9], and [10] are unrelated to the effects of this medication.

  1. Checking thefrequency and consistencyof bowel movements
  2. 34.) A client with Crohn's disease is

Rationale: scheduled to receive an infusion of

The principal manifestations of Crohn's disease are diarrhea and abdominal pain. infliximab (Remicade). The nurse assisting in

Infliximab (Remicade) is an immunomodulator that reduces the degree of caring for the client should take which action inflammation in the colon, thereby reducing the diarrhea. Options 1, 3, and 4 are to monitor the effectiveness of treatment?

  1. Monitoring the leukocyte count for 2 days unrelated to this medication. after the infusion
  2. Checking the frequency and consistency of bowel movements        
  3. Checking serum liver enzyme levels before and after the infusion  
  4. Carrying out a Hematest on gastric fluids after the infusion is completed

35. 35.) The client has a PRN prescription for loperamide hydrochloride (Imodium). The nurse understands that this medication is used for which condition?

1.         Constipation

2.         Abdominal pain

3.         An episode of diarrhea       

4.         Hematest-positive nasogastric tube            drainage              

3.An episode of diarrhea Rationale:

Loperamide is an antidiarrheal agent. It is used to manage acute and also chronic diarrhea in conditions such as inflammatory bowel disease. Loperamide also can be used to reduce the volume of drainage from an ileostomy. It is not used for the conditions in options 1, 2, and 4.

36. 36.) The client has a PRN prescription for ondansetron (Zofran). For which condition should this medication be administered to the postoperative client?

1.  Paralytic ileus

2.  Incisional pain             

3.  Urinary retention             

4.  Nausea and vomiting             

4.Nausea and vomiting Rationale:

Ondansetron is an antiemetic used to treat postoperative nausea and vomiting, as well as nausea and vomiting associated with chemotherapy. The other options are incorrect.

  1. Reduction of steatorrhea
  2. 37.) The client has begun medication therapy

Rationale: with pancrelipase (Pancrease MT). The nurse

Pancrelipase (Pancrease MT) is a pancreatic enzyme used in clients with pancreatitis evaluates that the medication is having the as a digestive aid. The medication should reduce the amount of fatty stools optimal intended benefit if which effect is

(steatorrhea). Another intended effect could be improved nutritional status. It is not observed?

  1. Weight loss used to treat abdominal pain or heartburn. Its use could result in weight gain but
  2. Relief of heartburnshould not result in weight loss if it is aiding in digestion.
  3. Reduction of steatorrhea  
  4. Absence of abdominal pain  

38. 38.) An older client recently has been taking cimetidine (Tagamet). The nurse monitors the client for which most frequent central nervous system side effect of this medication? 1. Tremors

2.  Dizziness

3.  Confusion

4.  Hallucinations

3.Confusion Rationale:

Cimetidine is a histamine 2 (H2)-receptor antagonist. Older clients are especially susceptible to central nervous system side effects of cimetidine. The most frequent of these is confusion. Less common central nervous system side effects include headache, dizziness, drowsiness, and hallucinations.

  1. One hour before mealsand at bedtime
  2. ) The client with a gastric ulcer has a prescription for

Rationale: sucralfate (Carafate), 1 g by mouth four times daily. The nurse

Sucralfate is a gastric protectant. The medication should be schedules the medication for which times?

  1. With meals and at bedtime scheduled for administration 1 hour before meals and at bedtime.
  2. Every 6 hours around the clock The medication is timed to allow it to form a protective coating
  3. One hour after meals and at bedtime over the ulcer before food intake stimulates gastric acid
  4. One hour before meals and at bedtime production and mechanical irritation. The other options are incorrect.

 

  1. Relief of epigastric pain
  1. ) The client who chronically uses nonsteroidal anti-

Rationale: inflammatory drugs has been taking misoprostol (Cytotec).

The client who chronically uses nonsteroidal anti-inflammatory

The nurse determines that the medication is having the drugs (NSAIDs) is prone to gastric mucosal injury. Misoprostol is a intended therapeutic effect if which of the following is gastric protectant and is given specifically to prevent this noted?

  1. Resolved diarrhea Diarrhea can be a side effect of the medication, but is
  2. Relief of epigastric pain not an intended effect. Options 3 and 4 are incorrect.
  3. Decreased platelet count  
  4. Decreased white blood cell count  
  5. Heartburn
  1. ) The client has been taking omeprazole (Prilosec) for 4

Rationale: weeks. The ambulatory care nurse evaluates that the client is

Omeprazole is a proton pump inhibitor classified as an antiulcer receiving optimal intended effect of the medication if the agent. The intended effect of the medication is relief of pain from client reports the absence of which symptom?

  1. Diarrhea gastric irritation, often called heartburn by clients. Omeprazole is
  2. Heartburn not used to treat the conditions identified in options 1, 3, and 4.
  3. Flatulence  
  4. Constipation  
  5. "The medications will kill the bacteria and stop the acid
  1. ) A client with a peptic ulcer is diagnosed with a

production."

Helicobacter pylori infection. The nurse is reinforcing

Rationale: teaching for the client about the medications prescribed,

Triple therapy for Helicobacter pylori infection usually includes including clarithromycin (Biaxin), esomeprazole (Nexium), and two antibacterial drugs and a proton pump inhibitor. amoxicillin (Amoxil). Which statement by the client indicates

Clarithromycin and amoxicillin are antibacterials. Esomeprazole is the best understanding of the medication regimen?

  1. "My ulcer will heal because these medications will kill the a proton pump inhibitor. These medications will kill the bacteria and decrease acid production. "
  2. "These medications are only taken when I have pain from my ulcer."     
  3. "The medications will kill the bacteria and stop the acid  "        
  4. "These medications will coat the ulcer and decrease the  acid production in my stomach."                

43. 43.) A histamine (H2)-receptor antagonist will be prescribed for a client. The nurse understands that which medications are H2-receptor antagonists? Select all that apply.

1.  Nizatidine (Axid)

2.  Ranitidine (Zantac)

3.  Famotidine (Pepcid)

4.  Cimetidine (Tagamet)

5.  Esomeprazole (Nexium)

6.  Lansoprazole  (Prevacid)

1. Nizatidine (Axid)

2. Ranitidine (Zantac)

3. Famotidine (Pepcid)4.Cimetidine (Tagamet) Rationale:

H2-receptor antagonists suppress secretion of gastric acid, alleviate symptoms of heartburn, and assist in preventing complications of peptic ulcer disease. These medications also suppress gastric acid secretions and are used in active ulcer disease, erosive esophagitis, and pathological hypersecretory conditions. The other medications listed are proton pump inhibitors.

H2-receptor antagonists medication names end with -dine.

Proton pump inhibitors medication names end with -zole.

  1. Suction equipment
  1. ) A client is receiving acetylcysteine

Rationale: (Mucomyst), 20% solution diluted in 0.9%

Acetylcysteine can be given orally or by nasogastric tube to treat acetaminophen normal saline by nebulizer. The nurse should overdose, or it may be given by inhalation for use as a mucolytic. The nurse have which item available for possible use administering this medication as a mucolytic should have suction equipment after giving this medication?

  1. Ambu bag available in case the client cannot manage to clear the increased volume of
  2. Intubation tray liquefied secretions.
  3. Nasogastric tube  
  4. Suction equipment  
    1. Take the tablet with a full glass of water.
  5. ) A client has a prescription to take

Rationale: guaifenesin (Humibid) every 4 hours, as

Guaifenesin is an expectorant. It should be taken with a full glass of water to needed. The nurse determines that the client decrease viscosity of secretions. Sustained-release preparations should not be understands the most effective use of this broken open, crushed, or chewed. The medication may occasionally cause medication if the client states that he or she dizziness, headache, or drowsiness as side effects. The client should contact the will:

  1. Watch for irritability as a side effect. health care provider if the cough lasts longer than 1 week or is accompanied by
  2. Take the tablet with a full glass of water. fever, rash, sore throat, or persistent headache.
  3. Take an extra dose if the cough is accompanied by fever.  
  4. Crush the sustained-release tablet if immediate relief is needed.  
  5. Sudden increase in pain
  1. ) A postoperative client has received a

Rationale: dose of naloxone hydrochloride for

Naloxone hydrochloride is an antidote to opioids and may also be given to the respiratory depression shortly after transfer postoperative client to treat respiratory depression. When given to the to the nursing unit from the postanesthesia postoperative client for respiratory depression, it may also reverse the effects of care unit. After administration of the analgesics. Therefore, the nurse must check the client for a sudden increase in the medication, the nurse checks the client for:

  1. Pupillary changes level of pain experienced. Options 1, 2, and 4 are not associated with this
  2. Scattered lung wheezes
  3. Sudden increase in pain  
  4. Sudden episodes of diarrhea  
    1. Peripheral neuritis
  5. ) A client has been taking isoniazid (INH) for

Rationale: 2 months. The client complains to a nurse

A common side effect of the TB drug INH is peripheral neuritis. This is manifested about numbness, paresthesias, and tingling in by numbness, tingling, and paresthesias in the extremities. This side effect can be the extremities. The nurse interprets that the minimized by pyridoxine (vitamin B6) intake. Options 1, 3, and 4 are incorrect. client is experiencing: 1. Hypercalcemia

  1. Peripheral neuritis
  2. Small blood vessel spasm
  3. Impaired peripheral circulation
  1. ) A client is to begin a 6-month course of therapy with isoniazid (INH). A nurse plans to teach the client to:
  2. Drink alcohol in small amounts only.
  3. Report yellow eyes or skin immediately.
  4. Increase intake of Swiss or aged cheeses.
  5. Avoid vitamin supplements during therapy.
  6. ) A client has been started on long-term therapy with rifampin (Rifadin). A nurse teaches the client that the medication:
    1. Should always be taken with food or antacids Should be double-dosed if one dose is forgotten 3. Causes orange discoloration of sweat, tears, urine, and feces 4. May be discontinued independently if symptoms are gone in 3 months
  7. ) A nurse has given a client taking ethambutol (Myambutol) information about the medication. The nurse determines that the client understands the instructions if the client states that he or she will immediately report: 1. Impaired sense of hearing
    1. Problems with visual acuity
    2. Gastrointestinal (GI) side effects Orange-red discoloration of body secretions
  8. ) Cycloserine (Seromycin) is added to the medication regimen for a client with tuberculosis. Which of the following would the nurse include in the client- teaching plan regarding this medication?
    1. To take the medication before meals
    2. To return to the clinic weekly for serum drug-level testing It is not necessary to call the health care provider (HCP) if a skin rash occurs.
    3. It is not necessary to restrict alcohol intake with this medication.
  9. Report yellow eyes or skin immediately.

Rationale:

INH is hepatotoxic, and therefore the client is taught to report signs and symptoms of hepatitis immediately (which include yellow skin and sclera). For the same reason, alcohol should be avoided during therapy. The client should avoid intake of Swiss cheese, fish such as tuna, and foods containing tyramine because they may cause a reaction characterized by redness and itching of the skin, flushing, sweating, tachycardia, headache, or lightheadedness. The client can avoid developing peripheral neuritis by increasing the intake of pyridoxine (vitamin B6) during the course of INH therapy for TB.

  1. Causes orange discoloration of sweat, tears, urine, and feces Rationale:

Rifampin should be taken exactly as directed as part of TB therapy. Doses should not be doubled or skipped. The client should not stop therapy until directed to do so by a health care provider. The medication should be administered on an empty stomach unless it causes gastrointestinal upset, and then it may be taken with food. Antacids, if prescribed, should be taken at least 1 hour before the medication. Rifampin causes orange-red discoloration of body secretions and will permanently stain soft contact lenses.

  1. Problems with visual acuity Rationale:

Ethambutol causes optic neuritis, which decreases visual acuity and the ability to discriminate between the colors red and green. This poses a potential safety hazard when a client is driving a motor vehicle. The client is taught to report this symptom immediately. The client is also taught to take the medication with food if GI upset occurs. Impaired hearing results from antitubercular therapy with streptomycin. Orange-red discoloration of secretions occurs with rifampin (Rifadin).

  1. To return to the clinic weekly for serum drug-level testing Rationale:

Cycloserine (Seromycin) is an antitubercular medication that requires weekly serum drug level determinations to monitor for the potential of neurotoxicity. Serum drug levels lower than 30 mcg/mL reduce the incidence of neurotoxicity. The medication must be taken after meals to prevent gastrointestinal irritation. The client must be instructed to notify the HCP if a skin rash or signs of central nervous system toxicity are noted. Alcohol must be avoided because it increases the risk of seizure activity.

  1. Liver enzyme levels
  2. ) A client with tuberculosis is being

Rationale: started on antituberculosis therapy with

INH therapy can cause an elevation of hepatic enzyme levels and hepatitis. isoniazid (INH). Before giving the client the

Therefore, liver enzyme levels are monitored when therapy is initiated and during first dose, a nurse ensures that which of the the first 3 months of therapy. They may be monitored longer in the client who is following baseline studies has been greater than age 50 or abuses alcohol. completed?

  1. Electrolyte levels  
  2. Coagulation times  
  3. Liver enzyme levels  
  4. Serum creatinine level  

1.Signs of hepatitis

  1. ) Rifabutin (Mycobutin) is prescribed for a
    1. Flu-like syndrome client with active Mycobacterium avium
    2. Low neutrophil count complex (MAC) disease and tuberculosis. The
    3. Ocular pain or blurred vision nurse monitors for which side effects of the

Rationale: medication? Select all that apply.

  1. Signs of hepatitis Rifabutin (Mycobutin) may be prescribed for a client with active MAC disease and
  2. Flu-like syndrome It inhibits mycobacterial DNA-dependent RNA polymerase and
  3. Low neutrophil count suppresses protein synthesis. Side effects include rash, gastrointestinal disturbances, Vitamin B6 deficiency neutropenia (low neutrophil count), red-orange body secretions, uveitis (blurred
  4. Ocular pain or blurred vision vision and eye pain), myositis, arthralgia, hepatitis, chest pain with dyspnea, and flu-
  5. Tingling and numbness of the fingers like syndrome. Vitamin B6 deficiency and numbness and tingling in the extremities are associated with the use of isoniazid (INH). Ethambutol (Myambutol) also causes peripheral neuritis.

 

  1. "I will take Ecotrin (enteric-coated aspirin) for my headaches because it is
  1. ) A nurse reinforces discharge instructions

coated." to a postoperative client who is taking

Rationale: warfarin sodium (Coumadin). Which

Ecotrin is an aspirin-containing product and should be avoided. Alcohol statement, if made by the client, reflects the consumption should be avoided by a client taking warfarin sodium. Taking need for further teaching?

  1. "I will take my pills every day at the same prescribed medication at the same time each day increases client compliance. The

Medic-Alert bracelet provides health care personnel emergency information.

time."

  1. "I will be certain to avoid alcohol  "     
  2. "I have already called my family to pick up a Medic-Alert bracelet."  
  3. "I will take Ecotrin (enteric-coated aspirin) for my headaches because it is coated."  

55. 55.) A client who is receiving digoxin (Lanoxin) daily has a serum potassium level of 3.0 mEq/L and is complaining of anorexia. A health care provider prescribes a digoxin level to rule out digoxin toxicity. A nurse checks the results, knowing that which of the following is the therapeutic serum level (range) for digoxin?

1.  3 to 5 ng/mL

2.  0.5 to 2  ng/mL

3.  1.2 to 2.8 ng/mL

4.  3.5 to 5.5 ng/mL

2.) 0.5 to 2 ng/mL Rationale:

Therapeutic levels for digoxin range from 0.5 to 2 ng/mL. Therefore, options 1, 3, and 4 are incorrect.

  1. ) Heparin sodium is prescribed for the client. The nurse expects that the health care provider will prescribe which of the following to monitor for a therapeutic effect of the medication? 1. Hematocrit level
    1. Hemoglobin level
    2. Prothrombin time (PT)
    3. Activated partial thromboplastin time

(aPTT)

  1. ) A nurse is monitoring a client who is taking propranolol (Inderal LA). Which data collection finding would indicate a potential serious complication associated with propranolol?
    1. The development of complaints of insomnia
    2. The development of audible expiratory wheezes
    3. A baseline blood pressure of 150/80 mm Hg followed by a blood pressure of 138/72 mm Hg after two doses of the medication
    4. A baseline resting heart rate of 88 beats/min followed by a resting heart rate of 72 beats/min after two doses of the medication
  2. ) Isosorbide mononitrate (Imdur) is prescribed for a client with angina pectoris. The client tells the nurse that the medication is causing a chronic headache. The nurse appropriately suggests that the client:
    1. Cut the dose in half.
    2. Discontinue the medication.
    3. Take the medication with food.
    4. Contact the health care provider (HCP).
  3. ) A client is diagnosed with an acute myocardial infarction and is receiving tissue plasminogen activator, alteplase (Activase, tPA). Which action is a priority nursing intervention?
    1. Monitor for renal failure.
    2. Monitor psychosocial status.
    3. Monitor for signs of bleeding.
    4. Have heparin sodium available.
  1. Activated partial thromboplastin time (aPTT) Rationale:

The PT will assess for the therapeutic effect of warfarin sodium (Coumadin) and the aPTT will assess the therapeutic effect of heparin sodium. Heparin sodium doses are determined based on these laboratory results. The hemoglobin and hematocrit values assess red blood cell concentrations.

  1. The development of audible expiratory wheezes Rationale:

Audible expiratory wheezes may indicate a serious adverse reaction, bronchospasm. β- Blockers may induce this reaction, particularly in clients with chronic obstructive pulmonary disease or asthma. Normal decreases in blood pressure and heart rate are expected. Insomnia is a frequent mild side effect and should be monitored.  

  1. Take the medication with food.

Rationale:

Isosorbide mononitrate is an antianginal medication. Headache is a frequent side effect of isosorbide mononitrate and usually disappears during continued therapy. If a headache occurs during therapy, the client should be instructed to take the medication with food or meals. It is not necessary to contact the HCP unless the headaches persist with therapy. It is not appropriate to instruct the client to discontinue therapy or adjust the dosages.

  1. Monitor for signs of bleeding.

Rationale:

Tissue plasminogen activator is a thrombolytic. Hemorrhage is a complication of any type of thrombolytic medication. The client is monitored for bleeding. Monitoring for renal failure and monitoring the client's psychosocial status are important but are not the most critical interventions. Heparin is given after thrombolytic therapy, but the question is not asking about follow-up medications.

  1. Hypokalemia, hyperglycemia, sulfaallergy
  2. ) A nurse is planning to administer

Rationale: hydrochlorothiazide (HydroDIURIL) to a

Thiazide diuretics such as hydrochlorothiazide are sulfa-based medications, and a client. The nurse understands that which of client with a sulfa allergy is at risk for an allergic reaction. Also, clients are at risk for the following are concerns related to the hypokalemia, hyperglycemia, hypercalcemia, hyperlipidemia, and hyperuricemia. administration of this medication?

  1. Hypouricemia, hyperkalemia
  2. Increased risk of osteoporosis Hypokalemia, hyperglycemia, sulfa allergy              
  3. Hyperkalemia, hypoglycemia, penicillin allergy           
  4. "I'll continue my nicotinic acid from the health food store."
  1. ) A home health care nurse is visiting a

Rationale: client with elevated triglyceride levels and

Nicotinic acid, even an over-the-counter form, should be avoided because it may a serum cholesterol level of 398 mg/dL. The lead to liver abnormalities. All lipid-lowering medications also can cause liver client is taking cholestyramine (Questran). abnormalities, so a combination of nicotinic acid and cholestyramine resin is to be

Which of the following statements, if made avoided. Constipation and bloating are the two most common side effects. Walking by the client, indicates the need for further and the reduction of fats in the diet are therapeutic measures to reduce cholesterol education?

  1. "Constipation and bloating might be a and triglyceride levels.

problem."                                                               

  1. "I'll continue to watch my diet and reduce my fats."  
  2. "Walking a mile each day will help the whole process."
  3. "I'll continue my nicotinic acid from the health food store."
  4. "Ibuprofen (Motrin) taken 30 minutes before the nicotinic acid should decrease the
  1. ) A client is on nicotinic acid (niacin) for

flushing." hyperlipidemia and the nurse provides

Rationale: instructions to the client about the

Flushing is a side effect of this medication. Aspirin or a nonsteroidal anti- medication. Which statement by the client inflammatory drug can be taken 30 minutes before taking the medication to decrease would indicate an understanding of the flushing. Alcohol consumption needs to be avoided because it will enhance this side instructions?

  1. "It is not necessary to avoid the use of The medication should be taken with meals, this will decrease gastrointestinal upset. Taking the medication with meals has no effect on the flushing. Clay-colored alcohol."
  2. "The medication should be taken with stools are a sign of hepatic dysfunction and should be immediately reported to the health care provider (HCP). meals to decrease flushing."
  3. "Clay-colored stools are a common side effect and should not be of concern." "Ibuprofen (Motrin) taken 30 minutes before the nicotinic acid should decrease the flushing."
  1. Contact the registered nurse.
  1. ) A client with coronary artery
    1. Assess the client's pain level. disease complains of substernal chest
    2. Check the client's blood pressure. After checking the client's heart
    3. Administer a second nitroglycerin, 0.4 mg, sublingually. rate and blood pressure, a nurse

Rationale: administers nitroglycerin, 0.4 mg,

The usual guideline for administering nitroglycerin tablets for a hospitalized client with sublingually. After 5 minutes, the client chest pain is to administer one tablet every 5 minutes PRN for chest pain, for a total dose states, "My chest still hurts." Select the of three tablets. The registered nurse should be notified of the client's condition, who will appropriate actions that the nurse then notify the health care provider as appropriate. Because the client is still complaining should take. Select all that apply.

  1. Call a code blue. of chest pain, the nurse would administer a second nitroglycerin tablet. The nurse would
  2. Contact the registered nurse. assess the client's pain level and check the client's blood pressure before administering
  3. Contact the client's family. each nitroglycerin dose. There are no data in the question that indicate the need to call a Assess the client's pain level. code blue. In addition, it is not necessary to contact the client's family unless the client
  4. Check the client's blood pressure. has requested this.
  5. Administer a second nitroglycerin,  

         0.4 mg, sublingually.                                    

  1. A decrease in the warfarin sodium (Coumadin) dosage
  1. ) Nalidixic acid (NegGram) is

Rationale: prescribed for a client with a urinary

Nalidixic acid can intensify the effects of oral anticoagulants by displacing these agents tract infection. On review of the client's from binding sites on plasma protein. When an oral anticoagulant is combined with record, the nurse notes that the client nalidixic acid, a decrease in the anticoagulant dosage may be needed. is taking warfarin sodium (Coumadin) daily. Which prescription should the  nurse anticipate for this client?  1. Discontinuation of warfarin sodium  

        (Coumadin)                                                   

  1. A decrease in the warfarin sodium  

        (Coumadin) dosage                                     

  1. An increase in the warfarin sodium

(Coumadin) dosage

  1. A decrease in the usual dose of

         nalidixic acid (NegGram)                           

  1. Maintain a high fluid intake.
  1. ) A nurse is reinforcing discharge

Rationale: instructions to a client receiving

Each dose of sulfisoxazole should be administered with a full glass of water, and the sulfisoxazole. Which of the following client should maintain a high fluid intake. The medication is more soluble in alkaline urine. should be included in the list of

The client should not be instructed to taper or discontinue the dose. Some forms of instructions?

  1. Restrict fluid intake. sulfisoxazole cause urine to turn dark brown or red. This does not indicate the need to
  2. Maintain a high fluid intake. notify the HCP.
  3. If the urine turns dark brown, call the health care provider (HCP) immediately.  
  4. Decrease the dosage when symptoms are improving to prevent an  allergic response.    
  5. Sore throat
  1. ) Trimethoprim-sulfamethoxazole

Rationale: (TMP-SMZ) is prescribed for a client. A

Clients taking trimethoprim-sulfamethoxazole (TMP-SMZ) should be informed about nurse should instruct the client to early signs of blood disorders that can occur from this medication. These include sore report which symptom if it developed throat, fever, and pallor, and the client should be instructed to notify the health care during the course of this medication provider if these symptoms occur. The other options do not require health care provider therapy?

  1. Nausea
  2. Diarrhea
  3. Headache
  4. Sore throat
  1. That a reddish orange discoloration ofthe urine may occur
  1. ) Phenazopyridine hydrochloride

Rationale: (Pyridium) is prescribed for a client for

The nurse should instruct the client that a reddish-orange discoloration of urine may symptomatic relief of pain resulting from occur. The nurse also should instruct the client that this discoloration can stain fabric. a lower urinary tract infection. The nurse

The medication should be taken after meals to reduce the possibility of gastrointestinal reinforces to the client:

  1. To take the medication at bedtime A headache is an occasional side effect of the medication and does not warrant
  2. To take the medication before meals discontinuation of the medication.
  3. To discontinue the medication if a headache occurs  
  4. That a reddish orange discoloration of the urine may occur  
  5. Urinary strictures
  1. ) Bethanechol chloride (Urecholine) is

Rationale: prescribed for a client with urinary

Bethanechol chloride (Urecholine) can be harmful to clients with urinary tract retention. Which disorder would be a obstruction or weakness of the bladder wall. The medication has the ability to contract contraindication to the administration of the bladder and thereby increase pressure within the urinary tract. Elevation of pressure this medication?

  1. Gastric atony within the urinary tract could rupture the bladder in clients with these conditions.
  2. Urinary strictures  
  3. Neurogenic atony
  4. Gastroesophageal reflux  
  5. Bradycardia
  1. ) A nurse who is administering

Rationale: bethanechol chloride (Urecholine) is

Toxicity (overdose) produces manifestations of excessive muscarinic stimulation such monitoring for acute toxicity associated as salivation, sweating, involuntary urination and defecation, bradycardia, and severe with the medication. The nurse checks the hypotension. Treatment includes supportive measures and the administration of client for which sign of toxicity?

  1. Dry skin atropine sulfate subcutaneously or intravenously.
  2. Dry mouth  
  3. Bradycardia  
  4. Signs of dehydration  
  5. Restlessness
  1. ) Oxybutynin chloride (Ditropan XL) is

Rationale: prescribed for a client with neurogenic

Toxicity (overdosage) of this medication produces central nervous system excitation, bladder. Which sign would indicate a such as nervousness, restlessness, hallucinations, and irritability. Other signs of toxicity possible toxic effect related to this include hypotension or hypertension, confusion, tachycardia, flushed or red face, and medication?

  1. Pallor signs of respiratory depression. Drowsiness is a frequent side effect of the medication
  2. Drowsiness but does not indicate overdosage.
  3. Bradycardia  
  4. Restlessness  
  5. Elevated blood urea nitrogen level
  1. ) After kidney transplantation,

Rationale: cyclosporine (Sand immune) is prescribed

Nephrotoxicity can occur from the use of cyclosporine (Sandimmune). Nephrotoxicity for a client. Which laboratory result would is evaluated by monitoring for elevated blood urea nitrogen (BUN) and serum indicate an adverse effect from the use of creatinine levels. Cyclosporine is an immunosuppressant but does not depress the

this medication?

  1. Decreased creatinine level bone marrow.
  2. Decreased hemoglobin level
  3. Elevated blood urea nitrogen level
  4. Decreased white blood cell count
  1. Renal insufficiency
  1. ) Cinoxacin (Cinobac), a urinary antiseptic, is prescribed

Rationale: for the client. The nurse reviews the client's medical

Cinoxacin should be administered with caution in clients with renal record and should contact the health care provider (HCP) impairment. The dosage should be reduced, and failure to do so could regarding which documented finding to verify the result in accumulation of cinoxacin to toxic levels. Therefore the nurse prescription? Refer to chart.

  1. Renal insufficiency would verify the prescription if the client had a documented history of 2. Chest x-ray: normal renal insufficiency. The laboratory and diagnostic test results are 3. Blood glucose, 102 mg/dL normal findings. Folic acid (vitamin B6) may be prescribed for a client 4. Folic acid (vitamin B6) 0.5 mg, orally dailywith renal insufficiency to prevent anemia.

73. 73.) A client with myasthenia gravis is suspected of having cholinergic crisis. Which of the following indicate that this crisis exists? 1. Ataxia

2.  Mouth sores

3.  Hypotension

4.  Hypertension

4. Hypertension Rationale:

Cholinergic crisis occurs as a result of an overdose of medication. Indications of cholinergic crisis include gastrointestinal disturbances, nausea, vomiting, diarrhea, abdominal cramps, increased salivation and tearing, miosis, hypertension, sweating, and increased bronchial secretions.

74. 74.) A client with myasthenia gravis is receiving pyridostigmine (Mestinon). The nurse monitors for signs and symptoms of cholinergic crisis caused by overdose of the medication. The nurse checks the medication supply to ensure that which medication is available for administration if a cholinergic crisis occurs?

1.  Vitamin K           

2.  Atropine sulfate             

3.  Protamine sulfate             

4.  Acetylcysteine (Mucomyst)      

2. Atropine sulfate Rationale:

The antidote for cholinergic crisis is atropine sulfate. Vitamin K is the antidote for warfarin (Coumadin). Protamine sulfate is the antidote for heparin, and acetylcysteine (Mucomyst) is the antidote for acetaminophen (Tylenol).

  1. A temporary worsening of the condition
  2. 75.) A client with myasthenia gravis becomes increasingly

Rationale:

weak. The health care provider prepares to identify

An edrophonium (Enlon) injection, a cholinergic drug, makes the client whether the client is reacting to an overdose of the in cholinergic crisis temporarily worse. This is known as a negative medication (cholinergic crisis) or increasing severity of the test. An improvement of weakness would occur if the client were

disease (myasthenic crisis). An injection of edrophonium experiencing myasthenia gravis. Options 1 and 2 would not occur in

(Enlon) is administered. Which of the following indicates either crisis. that the client is in cholinergic crisis?

  1. No change in the condition  
  2. Complaints of muscle spasms  
  3. An improvement of the weakness  
  4. A temporary worsening of the condition  
  5. Impaired voluntary movements
  6. ) Carbidopa-levodopa (Sinemet) is prescribed for a

Rationale:

client with Parkinson's disease, and the nurse monitors the

Dyskinesia and impaired voluntary movement may occur with high client for adverse reactions to the medication. Which of levodopa dosages. Nausea, anorexia, dizziness, orthostatic

the following indicates that the client is experiencing an hypotension, bradycardia, and akinesia (the temporary muscle adverse reaction?

  1. Pruritus weakness that lasts 1 minute to 1 hour, also known as the "on-off
  2. Tachycardia phenomenon") are frequent side effects of the medication.
  3. Hypertension
  4. Impaired voluntary movements
  1. ) Phenytoin (Dilantin), 100 mg orally three times daily, has been prescribed for a client for seizure control. The nurse reinforces instructions regarding the medication to the client. Which statement by the client indicates an understanding of the instructions?
  2. "I will use a soft toothbrush to brush my teeth."
  3. "It's all right to break the capsules to make it easier for me to swallow them." "If I forget to take my medication, I can wait until the next dose and eliminate that dose."

  1. "If my throat becomes sore, it's a normal effect of the medication and it's nothing to be concerned about."
  2. ) A client is taking phenytoin (Dilantin) for seizure control and a sample for a serum drug level is drawn. Which of the following indicates a therapeutic serum drug range?
    1. 5 to 10 mcg/mL
    2. 10 to 20 mcg/mL
    3. 20 to 30 mcg/mL
    4. 30 to 40 mcg/mL
  3. ) Ibuprofen (Advil) is prescribed for a client. The nurse tells the client to take the medication:
    1. With 8 oz of milk
    2. In the morning after arising
    3. 60 minutes before breakfast
    4. At bedtime on an empty stomach
  4. ) A nurse is caring for a client who is taking phenytoin (Dilantin) for control of seizures. During data collection, the nurse notes that the client is taking birth control pills. Which of the following information should the nurse provide to the client?
    1. Pregnancy should be avoided while taking phenytoin (Dilantin).
    2. The client may stop taking the phenytoin (Dilantin) if it is causing severe gastrointestinal effects. The potential for decreased effectiveness of the birth control pills exists while taking phenytoin (Dilantin).
    3. The increased risk of thrombophlebitis exists while taking phenytoin (Dilantin) and birth control pills together.
  5. "I will use a soft toothbrush to brush myteeth." Rationale:

Phenytoin (Dilantin) is an anticonvulsant. Gingival hyperplasia, bleeding, swelling, and tenderness of the gums can occur with the use of this medication. The client needs to be taught good oral hygiene, gum massage, and the need for regular dentist visits. The client should not skip medication doses, because this could precipitate a seizure.Capsules should not be chewed or broken and they must be swallowed. The client needs to be instructed to report a sore throat, fever, glandular swelling, or any skin reaction, because this indicates hematological toxicity.

  1. 10 to 20 mcg/mL Rationale:

The therapeutic serum drug level range for phenytoin (Dilantin) is 10 to 20 mcg/mL. * A helpful hint may be to remember that the theophylline therapeutic range and the acetaminophen (Tylenol) therapeutic range are the same as the phenytoin (Dilantin) therapeutic range.*

  1. With 8 oz of milk Rationale:

Ibuprofen is a nonsteroidal anti-inflammatory drug (NSAID). NSAIDs should be given with milk or food to prevent gastrointestinal irritation. Options 2, [11], and 4 are incorrect.

  1. ) A client with trigeminal neuralgia is being treated with carbamazepine (Tegretol). Which laboratory result would indicate that the client is experiencing an adverse reaction to the medication? 1. Sodium level, 140 mEq/L
  2. Uric acid level, 5.0 mg/dL 3. White blood cell count, 3000 cells/mm3
  3. Blood urea nitrogen (BUN) level, 15 mg/dL
  4. ) A client is receiving meperidine hydrochloride (Demerol) for pain. Which of the following are side effects of this medication. Select all that apply.
    1. Diarrhea
    2. Tremors
    3. Drowsiness
    4. Hypotension
    5. Urinary frequency
    6. Increased respiratory rate
  5. ) The client has been on treatment for rheumatoid arthritis for 3 weeks. During the administration of etanercept (Enbrel), it is most important for the nurse to check:
    1. The injection site for itching and

edema

  1. The white blood cell counts and platelet counts
  2. Whether the client is experiencing fatigue and joint pain
  3. A metallic taste in the mouth, with a loss of appetite
  4. White blood cell count, 3000 cells/mm3 Rationale:

Adverse effects of carbamazepine (Tegretol) appear as blood dyscrasias, including aplastic anemia, agranulocytosis, thrombocytopenia, leukopenia, cardiovascular disturbances, thrombophlebitis, dysrhythmias, and dermatological effects. Options 1, 2, and 4 identify normal laboratory values.

  1. Tremors
  2. DrowsinessHypotension Rationale:

Meperidine hydrochloride is an opioid analgesic. Side effects include respiratory depression, drowsiness, hypotension, constipation, urinary retention, nausea, vomiting, and tremors.

  1. The white blood cell counts and platelet counts Rationale:

Infection and pancytopenia are side effects of etanercept (Enbrel). Laboratory studies are performed before and during drug treatment. The appearance of abnormal white blood cell counts and abnormal platelet counts can alert the nurse to a potentially life- threatening infection. Injection site itching is a common occurrence following administration. A metallic taste with loss of appetite are not common signs of side effects of this medication.

84. 84.) Baclofen (Lioresal) is prescribed for the client with multiple sclerosis. The nurse assists in planning care, knowing that the primary therapeutic effect of this medication is which of the following?

1.  Increased muscle tone

2.  Decreased muscle spasms

3.  Increased range of motion

4.  Decreased local pain and tenderness

2.Decreased muscle spasms Rationale:

Baclofen is a skeletal muscle relaxant and central nervous system depressant and acts at the spinal cord level to decrease the frequency and amplitude of muscle spasms in clients with spinal cord injuries or diseases and in clients with multiple sclerosis. Options 1, 3, and 4 are incorrect.

  1. Drowsiness
  2. ) A nurse is monitoring a client

Rationale: receiving baclofen (Lioresal) for side

Baclofen is a central nervous system (CNS) depressant and frequently causes effects related to the medication. Which drowsiness, dizziness, weakness, and fatigue. It can also cause nausea, constipation, of the following would indicate that the and urinary retention. Clients should be warned about the possible reactions. Options 1, client is experiencing a side effect?

  1. Polyuria 2, and 4 are not side effects.
  2. Diarrhea  
  3. Drowsiness  
  4. Muscular excitability  
    1. Avoid the use of alcohol.
  5. ) A nurse is reinforcing discharge

Rationale: instructions to a client receiving baclofen

Baclofen is a central nervous system (CNS) depressant. The client should be cautioned

(Lioresal). Which of the following would against the use of alcohol and other CNS depressants, because baclofen potentiates the nurse include in the instructions?

  1. Restrict fluid intake. the depressant activity of these agents. Constipation rather than diarrhea is an adverse
  2. Avoid the use of alcohol. effect of baclofen. It is not necessary to restrict fluids, but the client should be warned
  3. Stop the medication if diarrhea occurs. that urinary retention can occur. Fatigue is related to a CNS effect that is most intense
  4. Notify the health care provider if during the early phase of therapy and diminishes with continued medication use. It is not necessary that the client notify the health care provider if fatigue occurs. fatigue occurs.
  5. "Weakness and fatigue commonly occur and will diminish with continued medication
  1. ) A client with acute muscle spasms has

use." been taking baclofen  (Lioresal).  The

Rationale: client calls the clinic nurse because of

The client should be instructed that symptoms such as drowsiness, weakness, and continuous feelings of weakness and fatigue are more intense in the early phase of therapy and diminish with continued fatigue and asks the nurse about medication use. The client should be instructed never to withdraw or stop the discontinuing the medication. The nurse medication abruptly, because abrupt withdrawal can cause visual hallucinations, should make which appropriate response paranoid ideation, and seizures. It is best for the nurse to inform the client that these to the client?

  1. "You should never stop the medication." symptoms will subside and encourage the client to continue the use of the medication.
  2. "It is best that you taper the dose if you intend to stop the medication."  
  3. "It is okay to stop the medication if you think that you can tolerate the muscle  "    
  4. "Weakness and fatigue commonly  occur and will diminish with continued

medication use."

  1. Acts directly on the skeletal muscle to relieve spasticity
  1. ) Dantrolene sodium (Dantrium) is

Rationale: prescribed for a client experiencing

Dantrium acts directly on skeletal muscle to relieve muscle spasticity. The primary flexor spasms, and the client asks the action is the suppression of calcium release from the sarcoplasmic reticulum. This in nurse about the action of the medication. turn decreases the ability of the skeletal muscle to contract.

The nurse responds, knowing that the

*Options 1, 3, and 4 are all comparable or alike in that they address CNS therapeutic action of this medication is suppression and the depression of reflexes. Therefore, eliminate these options.* which of the following?

  1. Depresses spinal reflexes
  2. Acts directly on the skeletal muscle to relieve spasticity
  3. Acts within the spinal cord to suppress hyperactive reflexes
  4. Acts on the central nervous system

(CNS) to suppress spasms

  1. Liver function tests
  2. ) A nurse is reviewing the laboratory studies on a

Rationale: client receiving dantrolene sodium (Dantrium).

Dose-related liver damage is the most serious adverse effect of dantrolene. Which laboratory test would identify an adverse

To reduce the risk of liver damage, liver function tests should be performed effect associated with the administration of this before treatment and periodically throughout the treatment course. It is medication?

  1. Creatinine administered in the lowest effective dosage for the shortest time necessary.
  2. Liver function tests *Eliminate options 1 and 3 because these tests both assess kidney
  3. Blood urea nitrogen *
  4. Hematological function tests  
  5. Seizure disorders
  1. ) A nurse is reviewing the record of a client who

Rationale: has been prescribed baclofen (Lioresal). Which of

Clients with seizure disorders may have a lowered seizure threshold when the following disorders, if noted in the client's baclofen is administered. Concurrent therapy may require an increase in the history, would alert the nurse to contact the health anticonvulsive medication. The disorders in options 2, 3, and 4 are not a care provider?

  1. Seizure disorders concern when the client is taking baclofen.
  2. Hyperthyroidism  
  3. Diabetes mellitus  
  4. Coronary artery disease  
  5. Glaucoma
  1. ) Cyclobenzaprine (Flexeril) is prescribed for a

Rationale: client to treat muscle spasms, and the nurse is

Because this medication has anticholinergic effects, it should be used with reviewing the client's record. Which of the following caution in clients with a history of urinary retention, angle-closure glaucoma, disorders, if noted in the client's record, would and increased intraocular pressure. Cyclobenzaprine hydrochloride should indicate a need to contact the health care provider be used only for short-term 2- to 3-week therapy. regarding the administration of this medication?

  1. Glaucoma  
  2. Emphysema  
  3. Hyperthyroidism  
  4. Diabetes mellitus  

1.Symptom control during periods of emotional stress

  1. ) In monitoring a client's response to disease-
    1. Normal white blood cell counts, platelet, and neutrophil counts modifying antirheumatic drugs (DMARDs), which
    2. Radiological findings that show nonprogression of joint degeneration findings would the nurse interpret as acceptable
    3. An increased range of motion in the affected joints 3 months into therapy responses? Select all that apply.
  2. Symptom control during periods of emotional Rationale:

Because emotional stress frequently exacerbates the symptoms of stress

  1. Normal white blood cell counts, platelet, and rheumatoid arthritis, the absence of symptoms is a positive finding. DMARDs are given to slow progression of joint degeneration. In addition, the neutrophil counts
  2. Radiological findings that show nonprogression of improvement in the range of motion after 3 months of therapy with normal blood work is a positive finding. Temperature elevation and inflammation joint degeneration
  3. An increased range of motion in the affected and irritation at the medication injection site could indicate signs of infection. joints 3 months into therapy
  4. Inflammation and irritation at the injection site 3 days after injection is given  
  5. A low-grade temperature upon rising in the morning that remains throughout the day  
  6. Gait
  1. ) The client who is human immunodeficiency virus

Rationale: seropositive has been taking stavudine (d4t, Zerit).

Stavudine (d4t, Zerit) is an antiretroviral used to manage human

The nurse monitors which of the following most immunodeficiency virus infection in clients who do not respond to or who closely while the client is taking this medication?

  1. Gait cannot tolerate conventional therapy. The medication can cause peripheral
  2. Appetite neuropathy, and the nurse should monitor the client's gait closely and ask Level of consciousness        the client about paresthesia. Options 2, 3, and 4 are unrelated to the use of
  3. Hemoglobin and hematocrit blood levels the medication.
  1. Complete blood count
  1. ) The client with acquired immunodeficiency

Rationale: syndrome has begun therapy with zidovudine

A common side effect of therapy with zidovudine is leukopenia and anemia.

(Retrovir, Azidothymidine, AZT, ZDV). The nurse

The nurse monitors the complete blood count results for these changes. carefully monitors which of the following

Options 1, 2, and 3 are unrelated to the use of this medication. laboratory results during treatment with this medication?         

  1. Blood culture  
  2. Blood glucose level  
  3. Blood urea nitrogen  
  4. Complete blood count  
  5. Serum amylase
  1. ) The nurse is reviewing the results of serum

Rationale: laboratory studies drawn on a client with

Didanosine (Videx) can cause pancreatitis. A serum amylase level that is acquired immunodeficiency syndrome who is increased 1.5 to 2 times normal may signify pancreatitis in the client with receiving didanosine (Videx). The nurse interprets acquired immunodeficiency syndrome and is potentially fatal. The medication that the client may have the medication may have to be discontinued. The medication is also hepatotoxic and can discontinued by the health care provider if which

result in liver failure. of the following significantly elevated results is noted?  

  1. Serum protein  
  2. Blood glucose  
  3. Serum amylase  
  4. Serum creatinine  
  5. Blood pressure
  1. ) The nurse is caring for a postrenal transplant

Rationale: client taking cyclosporine (Sandimmune, Gengraf,

Hypertension can occur in a client taking cyclosporine (Sandimmune, Gengraf,

Neoral). The nurse notes an increase in one of

Neoral), and because this client is also complaining of a headache, the blood the client's vital signs, and the client is pressure is the vital sign to be monitoring most closely. Other adverse effects complaining of a headache. What is the vital sign include infection, nephrotoxicity, and hirsutism. Options 1, 2, and 4 are unrelated that is most likely increased?

  1. Pulse to the use of this medication.
  2. Respirations  
  3. Blood pressure  
  4. Pulse oximetry  
  5. Hearing loss
  1. ) Amikacin (Amikin) is prescribed for a client

Rationale: with a bacterial infection. The client is instructed

Amikacin (Amikin) is an aminoglycoside. Adverse effects of aminoglycosides to contact the health care provider (HCP) include ototoxicity (hearing problems), confusion, disorientation, immediately if which of the following occurs?

  1. Nausea gastrointestinal irritation, palpitations, blood pressure changes, nephrotoxicity,
  2. Lethargy and hypersensitivity. The nurse instructs the client to report hearing loss to the
  3. Hearing loss HCP immediately. Lethargy and muscle aches are not associated with the use
  4. Muscle aches of this medication. It is not necessary to contact the HCP immediately if nausea If nausea persists or results in vomiting, the HCP should be notified.

*(most aminoglycoside medication names end in the letters -cin)*

 

  1. Serum creatinine
  1. ) The nurse is assigned to care for a client with

Rationale: cytomegalovirus retinitis and acquired

Foscarnet is toxic to the kidneys. Serum creatinine is monitored before therapy, immunodeficiency syndrome who is receiving two to three times per week during induction therapy, and at least weekly

foscarnet. The nurse should check the latest during maintenance therapy. Foscarnet may also cause decreased levels of results of which of the following laboratory calcium, magnesium, phosphorus, and potassium. Thus these levels are also studies while the client is taking this medication?

  1. CD4 cell count measured with the same frequency.
  2. Serum albumin
  3. Serum creatinine
  4. Lymphocyte count
  1. The result ofanother infection caused by leukopenic effects of the medication.
  1. ) The client with acquired immunodeficiency

Rationale: syndrome and Pneumocystis jiroveci infection

Frequent side effects of this medication include leukopenia, thrombocytopenia, has been receiving pentamidine isethionate and anemia. The client should be monitored routinely for signs and symptoms of

(Pentam 300). The client develops a infection. Options 1, 2, and 3 are inaccurate interpretations. temperature of 101° F. The nurse does further

monitoring of the client, knowing that this sign         would most likely indicate:  

  1. The dose of the medication is too low. The client is experiencing toxic effects of the  medication.             
  2. The client has developed inadequacy of  
  1. The result of another infection caused by

leukopenic effects of the medication.

  1. Avoid sun exposure.
  1. ) Saquinavir (Invirase) is prescribed for the

Rationale: client who is human immunodeficiency virus

Saquinavir (Invirase) is an antiretroviral (protease inhibitor) used with other seropositive. The nurse reinforces medication antiretroviral medications to manage human immunodeficiency virus infection. instructions and tells the client to:

  1. Avoid sun exposure. Saquinavir is administered with meals and is best absorbed if the client Eat low-calorie foods. consumes high-calorie, high-fat meals. Saquinavir can cause photosensitivity, and
  2. Eat foods that are low in fat. the nurse should instruct the client to avoid sun exposure.
  3. Take the medication on an empty stomach.

2.Instruct the client to avoid alcohol.

  1. ) Ketoconazole is prescribed for a client
  2. Monitor hepatic and liver function studies. with a diagnosis of candidiasis. Select the
  3. Instruct the client to avoid exposure to the sun. interventions that the nurse includes when

Rationale: administering this medication. Select all that

Ketoconazole is an antifungal medication. It is administered with food (not on an apply.

  1. Restrict fluid intake. empty stomach) and antacids are avoided for 2 hours after taking the medication
  2. Instruct the client to avoid alcohol. to ensure absorption. The medication is hepatotoxic and the nurse monitors liver
  3. Monitor hepatic and liver function studies. function studies. The client is instructed to avoid exposure to the sun because the
  4. Administer the medication with an antacid. medication increases photosensitivity. The client is also instructed to avoid
  5. Instruct the client to avoid exposure to the There is no reason for the client to restrict fluid intake. In fact, this could be harmful to the client. sun.
  6. Administer the medication on an empty                

1.Rash

  1. ) A client with human immunodeficiency
  2. Hepatotoxicity virus is taking nevirapine (Viramune). The nurse

Rationale: should monitor for which adverse effects of the

Nevirapine (Viramune) is a non-nucleoside reverse transcriptase inhibitors (NRTI) medication? Select all that apply.

  1. Rash that is used to treat HIV infection. It is used in combination with other
  2. Hepatotoxicity antiretroviral medications to treat HIV. Adverse effects include rash, Stevens- Hyperglycemia Johnson syndrome, hepatitis, and increased transaminase levels. Hyperglycemia,
  3. Peripheral neuropathy peripheral neuropathy, and reduced bone density are not adverse effects of this
  4. Reduced bone mineral density
  1. White blood cell count
  1. ) A nurse is caring for a hospitalized

Rationale: client who has been taking clozapine

Hematological reactions can occur in the client taking clozapine and include

(Clozaril) for the treatment of a agranulocytosis and mild leukopenia. The white blood cell count should be checked schizophrenic disorder. Which laboratory before initiating treatment and should be monitored closely during the use of this study prescribed for the client will the medication. The client should also be monitored for signs indicating agranulocytosis, nurse specifically review to monitor for an which may include sore throat, malaise, and fever. Options 1, 2, and 4 are unrelated to adverse effect associated with the use of this medication. this medication?

  1. Platelet count  
  2. Cholesterol level  
  3. White blood cell count  
  4. Blood urea nitrogen level  
  5. When the last alcoholic drink was consumed
  6. ) Disulfiram (Antabuse) is prescribed for

Rationale: a client who is seen in the psychiatric

Disulfiram is used as an adjunct treatment for selected clients with chronic health care clinic. The nurse is collecting alcoholism who want to remain in a state of enforced sobriety. Clients must abstain data on the client and is providing from alcohol intake for at least 12 hours before the initial dose of the medication is instructions regarding the use of this administered. The most important data are to determine when the last alcoholic drink medication. Which is most important for the was consumed. The medication is used with caution in clients with diabetes mellitus, nurse to determine  before  administration hypothyroidism, epilepsy, cerebral damage, nephritis, and hepatic disease. It is also of this medication?

  1. A history of hyperthyroidism contraindicated in severe heart disease, psychosis, or hypersensitivity related to the
  2. A history of diabetes insipidus
  3. When the last full meal was consumed When the last alcoholic drink was  consumed  
  4. Dementia
  1. ) A nurse is collecting data from a

Rationale: client and the client's spouse reports that

Donepezil hydrochloride is a cholinergic agent used in the treatment of mild to the client is taking donepezil hydrochloride moderate dementia of the Alzheimer type. It enhances cholinergic functions by

(Aricept). Which disorder would the nurse increasing the concentration of acetylcholine. It slows the progression of Alzheimer's suspect that this client may have based on disease. Options 2, 3, and 4 are incorrect. the use of this medication?

  1. Dementia  
  2. Schizophrenia  
  3. Seizure disorder  
  4. Obsessive-compulsive disorder  
  5. "I should take the medication in the morning when I first arise." 106. 106.) Fluoxetine (Prozac) is prescribed for

Rationale: the client. The nurse reinforces instructions

Fluoxetine hydrochloride is administered in the early morning without consideration to the client regarding  the  administration

to meals. of the medication. Which statement by the

*Eliminate options 1, 2, and 4 because they are comparable or alike and indicate client indicates an understanding about taking the medication with an antacid or food.* administration of the medication?

  1. "I should take the medication with my evening meal."
  2. "I should take the medication at noon with an antacid."
  3. "I should take the medication in the morning when I first arise."
  4. "I should take the medication right before bedtime with a snack."

3.Arrives at the clinic neat and appropriate in appearance

  1. ) A client receiving a tricyclic antidepressant arrives at

Rationale: the mental health clinic. Which observation indicates that

Depressed individuals will sleep for long periods, are not able to go the client is correctly following the medication plan?

  1. Reports not going to work for this past week to work, and feel as if they cannot "do anything." Once they have had
  2. Complains of not being able to "do anything" anymore some therapeutic effect from their medication, they will report Arrives at the clinic neat and appropriate in appearance resolution of many of these complaints as well as demonstrate an
  3. Reports sleeping 12 hours per night and 3 to 4 hours improvement in their appearance. during the day
  4. Gastrointestinal dysfunctions
  1. ) A nurse is performing a follow-up teaching session

Rationale: with a client discharged 1 month ago who is taking

The most common adverse effects related to fluoxetine include fluoxetine (Prozac). What information would be important central nervous system (CNS) and gastrointestinal (GI) system for the nurse to gather regarding the adverse effects dysfunction. This medication affects the GI system by causing nausea related to the medication?

  1. Cardiovascular symptoms and vomiting, cramping, and diarrhea. Options 1, 3, and 4 are not
  2. Gastrointestinal dysfunctions adverse effects of this medication.
  3. Problems with mouth dryness  
  4. Problems with excessive sweating  
  5. No rapid heartbeats or anxiety
  1. ) A client taking buspirone (BuSpar) for 1 month returns

Rationale: to the clinic for a follow-up visit. Which of the following

Buspirone hydrochloride is not recommended for the treatment of would indicate medication effectiveness?

  1. No rapid heartbeats or anxiety drug or alcohol withdrawal, paranoid thought disorders, or
  2. No paranoid thought processes schizophrenia (thought broadcasting or delusions). Buspirone No thought broadcasting or delusions hydrochloride is most often indicated for the treatment of anxiety 4. No reports of alcohol withdrawal symptoms and aggression.
  3. Toxic
  1. ) A client taking lithium carbonate (Lithobid) reports

Rationale: vomiting, abdominal pain, diarrhea, blurred vision, tinnitus,

The therapeutic serum level of lithium is 0.6 to 1.2 mEq/L. A level of and tremors. The lithium level is checked as a part of the

3 mEq/L indicates toxicity. routine follow-up and the level is 3.0 mEq/L. The nurse knows that this level is:         

  1. Toxic  
  2. Normal  
  3. Slightly above normal  
  4. Excessively below normal  
  5. Seizure activity
  1. ) A client arrives at the health care clinic and tells the

Rationale: nurse that he has been doubling his daily dosage of

Bupropion does not cause significant orthostatic blood pressure bupropion hydrochloride (Wellbutrin) to help him get changes. Seizure activity is common in dosages greater than 450 mg

better faster. The nurse understands that the client is now daily. Bupropion frequently causes a drop in body weight. Insomnia at risk for which of the following?

  1. Insomniais a side effect, but seizure activity causes a greater client risk.
  2. Weight gain
  3. Seizure activity
  4. Orthostatic hypotension
  1. Figs
  1. ) A hospitalized client is started on
  2. Yogurt phenelzine sulfate (Nardil) for the treatment
  3. Aged cheese of depression. The nurse instructs the client

Rationale: to avoid consuming which foods while taking

Phenelzine sulfate (Nardil) is a monoamine oxidase inhibitor(MAOI). The client this medication? Select all that apply.

  1. Figs should avoid taking in foods that are high in tyramine. Use of these foods could
  2. Yogurt trigger a potentially fatal hypertensive crisis. Some foods to avoid include yogurt,
  3. Crackers aged cheeses, smoked or processed meats, red wines, and fruits such as
  4. Aged cheese avocados, raisins, and figs.
  5. Tossed salad  
  6. Oatmeal cookies  
    1. Maintain a high fluid intake.
  7. ) A nurse is reinforcing discharge

Rationale: instructions to a client receiving sulfisoxazole.

Each dose of sulfisoxazole should be administered with a full glass of water, and

Which of the following would be included in the client should maintain a high fluid intake. The medication is more soluble in the plan of care for instructions?

  1. Maintain a high fluid intake. alkaline urine. The client should not be instructed to taper or discontinue the dose.
  2. Discontinue the medication when feeling Some forms of sulfisoxazole cause the urine to turn dark brown or red. This does not indicate the need to notify the health care provider.
  3. If the urine turns dark brown, call the health care provider immediately.  
  4. Decrease the dosage when symptoms are improving to prevent an allergic response.     

114. 114.) A postoperative client requests medication for flatulence (gas pains). Which medication from the following PRN list should the nurse administer to this client?

1.  Ondansetron (Zofran)

2.  Simethicone (Mylicon)

3.  Acetaminophen (Tylenol)        

4.  Magnesium hydroxide (milk of magnesia,       

      MOM)                                                                        

2.Simethicone (Mylicon) Rationale:

Simethicone is an antiflatulent used in the relief of pain caused by excessive gas in the gastrointestinal tract. Ondansetron is used to treat postoperative nausea and vomiting. Acetaminophen is a nonopioid analgesic. Magnesium hydroxide is an antacid and laxative.

115. 115.) A client received 20 units of NPH insulin subcutaneously at 8:00 AM. The nurse should check the client for a potential hypoglycemic reaction at what time?

1.  5:00 PM

2.  10:00 AM             

3.  11:00 AM             

4.  11:00 PM             

1. 5:00 PMRationale:

NPH is intermediate-acting insulin. Its onset of action is 1 to 2½ hours, it peaks in 4 to 12 hours, and its duration of action is 24 hours. Hypoglycemic reactions most likely occur during peak time.

116. 116.) A nurse administers a dose of scopolamine (Transderm-Scop) to a postoperative client. The nurse tells the client to expect which of the following side effects of this medication?

1.  Dry mouth

2.  Diaphoresis

3.  Excessive urination

4.  Pupillary constriction

1.Dry mouth Rationale:

Scopolamine is an anticholinergic medication for the prevention of nausea and vomiting that causes the frequent side effects of dry mouth, urinary retention, decreased sweating, and dilation of the pupils. The other options describe the opposite effects of cholinergic-blocking agents and therefore are incorrect.

  1. Difficulty discriminating the color red from green
  2. 117.) A nurse has given the client taking

Rationale: ethambutol (Myambutol) information about the

Ethambutol causes optic neuritis, which decreases visual acuity and the ability to medication. The nurse determines that the discriminate between the colors red and green. This poses a potential safety client understands the instructions if the client hazard when driving a motor vehicle. The client is taught to report this symptom immediately reports:

  1. Impaired sense of hearing The client is also taught to take the medication with food if
  2. Distressing gastrointestinal side effects gastrointestinal upset occurs. Impaired hearing results from antitubercular Orange-red discoloration of body secretions therapy with streptomycin. Orange-red discoloration of secretions occurs with
  3. Difficulty discriminating the color red from rifampin (Rifadin). green  
  4. "I can change the time of my medication on the mornings that I feel strong." 118. 118.) A nurse is caring for an older client with a

Rationale: diagnosis of myasthenia gravis and has

The client with myasthenia gravis should be taught that timing of reinforced self-care instructions. Which anticholinesterase medication is critical. It is important to instruct the client to statement by the client indicates that further administer the medication on time to maintain a chemical balance at the teaching is necessary?

  1. "I rest each afternoon after my walk." neuromuscular junction. If not given on time, the client may become too weak to
  2. "I cough and deep breathe many times during Options 1, 2, and 3 include the necessary information that the client needs to understand to maintain health with this neurological degenerative the day."
  3. "If I get abdominal cramps and diarrhea, I

         should call my doctor."                                              

  1. "I can change the time of my medication on the mornings that I feel strong."  
  2. Low blood glucose level
  3. ) A client with diabetes mellitus who has

Rationale: been controlled with daily insulin has been β-Adrenergic blocking agents, such as atenolol, inhibit the appearance of signs placed on atenolol (Tenormin) for the control and symptoms of acute hypoglycemia, which would include nervousness, of angina pectoris. Because of the effects of increased heart rate, and sweating. Therefore, the client receiving this medication atenolol, the nurse determines that which of the should adhere to the therapeutic regimen and monitor blood glucose levels following is the most reliable indicator of carefully. Option 4 is the most reliable indicator of hypoglycemia. hypoglycemia?

  1. Sweating
  2. Tachycardia  
  3. Nervousness  
  4. Low blood glucose level  
  5. Acetaminophen (Tylenol)
  1. ) A client is taking lansoprazole (Prevacid)

Rationale: for the chronic management of Zollinger-

Zollinger-Ellison syndrome is a hypersecretory condition of the stomach. The

Ellison syndrome. The nurse advises the client client should avoid taking medications that are irritating to the stomach lining. to take which of the following products if

Irritants would include aspirin and nonsteroidal antiinflammatory drugs needed for a headache?

  1. Naprosyn (Aleve)(ibuprofen). The client should be advised to take acetaminophen for headache.
  2. Ibuprofen (Advil) *Remember that options that are comparable or alike are not likely to be
  3. Acetaminophen (Tylenol) With this in mind, eliminate options 1 and 2 first.*
  4. Acetylsalicylic acid (aspirin)
  1. Triamterene is a potassium-sparing diuretic, whereas hydrochlorothiazide is a
  1. ) A client who is taking

potassium-losing diuretic. hydrochlorothiazide (HydroDIURIL, HCTZ)

Rationale: has been started on triamterene

Potassium-sparing diuretics include amiloride (Midamor), spironolactone (Aldactone),

(Dyrenium) as well. The client asks the and triamterene (Dyrenium). They are weak diuretics that are used in combination with nurse why both medications are required. potassium-losing diuretics. This combination is useful when medication and dietary

The nurse formulates a response, based

on the understanding that:

1.                Both are weak potassium-losing diuretics.

2.                The combination of these medications prevents renal toxicity.

3.                Hydrochlorothiazide is an expensive medication, so using a combination of diuretics is cost-effective.

4.                Triamterene is a potassium-sparing

supplement of potassium is not appropriate. The use of two different diuretics does not prevent renal toxicity. Hydrochlorothiazide is an effective and inexpensive generic form of the thiazide classification of diuretics.

*It is especially helpful to remember that hydrochlorothiazide is a potassium- losing diuretic and triamterene is a potassium-sparing diuretic*

diuretic, whereas hydrochlorothiazide is a  potassium-losing diuretic.  

  1. Informing the client that impaired taste is expected and generally disappears in [12][13] to 122. 122.) A client who has begun taking

3 months fosinopril (Monopril) is very distressed,

Rationale: telling the nurse that he cannot taste food

ACE inhibitors, such as fosinopril, cause temporary impairment of taste (dysgeusia). normally since beginning the medication 2

The nurse can tell the client that this effect usually disappears in 2 to 3 months, even

weeks ago. The nurse provides the best

support to the client by:

1.                Telling the client not to take the medication with food

2.                Suggesting that the client taper the dose until taste returns to normal

3.                Informing the client that impaired taste is expected and generally disappears in 2 to

3 months

4. Requesting that the health care provider (HCP) change the prescription to another brand of angiotensin-converting enzyme (ACE) inhibitor

with continued therapy, and provide nutritional counseling if appropriate to avoid weight loss. Options [14], [15][16], and [17] are inappropriate actions. Taking this medication with or without food does not affect absorption and action. The dosage should never be tapered without HCP approval and the medication should never be stopped abruptly.

  1. Heart rate
  2. ) A nurse is preparing to administer

Rationale: digoxin (Lanoxin), 0.125 mg orally, to a

Digoxin is a cardiac glycoside that is used to treat heart failure and acts by client with heart failure. Which vital sign is increasing the force of myocardial contraction. Because bradycardia may be a most important for the nurse to check clinical sign of toxicity, the nurse counts the apical heart rate for 1 full minute before before administering the medication?

  1. Heart rate administering the medication. If the pulse rate is less than 60 beats/minute in an Temperature adult client, the nurse would withhold the medication and report the pulse rate to the
  2. Respirations registered nurse, who would then contact the health care provider.
  3. Blood pressure  

2.Tinnitus

  1. ) A nurse is caring for a client who has
    1. Hypotension been prescribed furosemide (Lasix) and is
    2. Hypokalemia monitoring for adverse effects associated

Rationale: with this medication. Which of the following

Furosemide is a loop diuretic; therefore, an expected effect is increased urinary should the nurse recognize as a potential frequency. Nausea is a frequent side effect, not an adverse effect. Photosensitivity is adverse effect Select all that apply.

  1. Nausea an occasional side effect. Adverse effects include tinnitus (ototoxicity), hypotension,
  2. Tinnitus and hypokalemia and occur as a result of sudden volume depletion.
  3. Hypotension  
  4. Hypokalemia  
  5. Photosensitivity  
  6. Increased urinary frequency  
  7. "I can skip a dose once a week."
  1. ) The nurse provides medication

Rationale: instructions to an older hypertensive client

Lisinopril is an antihypertensive angiotensin-converting enzyme (ACE) inhibitor. The who is taking 20 mg of lisinopril (Prinivil, usual dosage range is 20 to 40 mg per day. Adverse effects include headache,

Zestril) orally daily. The nurse evaluates the dizziness, fatigue, orthostatic hypotension, tachycardia, and angioedema. Specific need for further teaching when the client client teaching points include taking one pill a day, not stopping the medication states which of the following?

  1. "I can skip a dose once a week." without consulting the health care provider (HCP), and monitoring for side effects
  2. "I need to change my position slowly." and adverse reactions. The client should notify the HCP if side effects occur.
  3. "I take the pill after breakfast each day." "If I get a bad headache, I should call my  doctor immediately."          
    1. "I can't drink alcohol while I am taking my medication."
  4. ) A nurse is providing instructions to an

Rationale: adolescent who has a history of seizures

Alcohol will lower the seizure threshold and should be avoided. Adolescents can and is taking an anticonvulsant medication. obtain a driver's license in most states when they have been seizure free for 1 year.

Which of the following statements indicates

Anticonvulsants cause acne and oily skin; therefore a dermatologist may need to be that the client understands the consulted. If an anticonvulsant medication is missed, the health care provider should instructions?

  1. "I will never be able to drive a car." be notified.
  2. "My anticonvulsant medication will clear up my skin."
  3. "I can't drink alcohol while I am taking my medication."
  4. "If I forget my morning medication, I can take two pills at bedtime."
  1. Thrombophlebitis
  1. ) Megestrol acetate (Megace), an

Rationale: antineoplastic medication, is prescribed

Megestrol acetate (Megace) suppresses the release of luteinizing hormone from the for the client with metastatic endometrial anterior pituitary by inhibiting pituitary function and regressing tumor size. Megestrol is carcinoma. The nurse reviews the client's used with caution if the client has a history of thrombophlebitis. history and contacts the  registered nurse

*megestrol acetate is a hormonal antagonist enzyme and that a side effect is if which diagnosis is documented in the thrombotic disorders* client's history?

  1. Gout  
  2. Asthma  
  3. Thrombophlebitis  
  4. Myocardial infarction  
    1. Increased uric acid level
  5. ) The nurse is analyzing the laboratory

Rationale: results of a client with leukemia who has

Hyperuricemia is especially common following treatment for leukemias and received a regimen of chemotherapy. lymphomas because chemotherapy results in a massive cell kill. Although options 1, 2,

Which laboratory value would the nurse and 4 also may be noted, an increased uric acid level is related specifically to cell specifically note as a result of the massive

destruction. cell destruction that occurred from the chemotherapy?          

  1. Anemia  
  2. Decreased platelets  
  3. Increased uric acid level  
  4. Decreased leukocyte count  
  5. Increase fluid intake to 2000 to 3000 mL daily.
  6. ) The nurse is reinforcing medication

Rationale: instructions to a client with breast cancer

Hemorrhagic cystitis is a toxic effect that can occur with the use of who is receiving cyclophosphamide cyclophosphamide. The client needs to be instructed to drink copious amounts of (Neosar). The nurse tells the client to:

  1. Take the medication with food. fluid during the administration of this medication. Clients also should monitor urine
  2. Increase fluid intake to 2000 to 3000 output for hematuria. The medication should be taken on an empty stomach, unless gastrointestinal (GI) upset occurs. Hyperkalemia can result from the use of the mL daily.
  3. Decrease sodium intake while taking medication; therefore the client would not be told to increase potassium intake. The client would not be instructed to alter sodium intake. the medication.
  4. Increase potassium intake while taking the medication.            
  5. Crackles on auscultation of the lungs
  1. ) The client with non-Hodgkin's

Rationale: lymphoma is receiving daunorubicin

Cardiotoxicity noted by abnormal electrocardiographic findings or cardiomyopathy

(DaunoXome). Which of the following manifested as congestive heart failure is a toxic effect of daunorubicin. Bone marrow would indicate to the nurse that the client depression is also a toxic effect. Nausea and vomiting are frequent side effects is experiencing a toxic effect related to associated with the medication that begins a few hours after administration and lasts the medication?

  1. Fever 24 to 48 hours. Fever is a frequent side effect, and diarrhea can occur occasionally.
  2. Diarrhea The other options, however, are not toxic effects.
  3. Complaints of nausea and vomiting *keep in mind that the question is asking about a toxic effect and think: ABCs—
  4. Crackles on auscultation of the lungs airway, breathing, and circulation*
  5. Drowsiness
  1. ) A nurse is monitoring a client

Rationale:

receiving desmopressin acetate (DDAVP)

Water intoxication (overhydration) or hyponatremia is an adverse effect to for adverse effects to the medication. desmopressin. Early signs include drowsiness, listlessness, and headache. Decreased

Which of the following indicates the urination, rapid weight gain, confusion, seizures, and coma also may occur in

presence of an adverse effect?

  1. Insomnia
  2. Drowsiness *Recall that this medication is used to treat diabetes insipidus to eliminate weight 3. Weight loss loss and increased urination.*
  3. Increased urination

Rationale: 134. 134.) A nurse reinforces instructions to a client who is

Oral doses of levothyroxine (Synthroid) should be taken on an empty taking levothyroxine (Synthroid). The nurse tells the stomach to enhance absorption. Dosing should be done in the client to take the medication:

  1. With foodmorning before breakfast.
  2. At lunchtime *Note that options 1, 2, and 4 are comparable or alike in that these
  3. On an empty stomach options address administering the medication with food.*
  4. At bedtime with a snack  

135. 135.) A nurse reinforces medication instructions to a client who is taking levothyroxine  (Synthroid).  The  nurse instructs the client to notify the  health care  provider (HCP) if which of the following occurs?

1.  Fatigue

2.  Tremors

3.  Cold intolerance

4.  Excessively dry skin  

2. Tremors Rationale:

Excessive doses of levothyroxine (Synthroid) can produce signs and symptoms of hyperthyroidism. These include tachycardia, chest pain, tremors, nervousness, insomnia, hyperthermia, heat intolerance, and sweating. The client should be instructed to notify the HCP if these occur. Options 1, 3, and 4 are signs of hypothyroidism.

136. 136.) A nurse performs an admission assessment on a client who visits a health care clinic for the first time. The client tells the nurse that propylthiouracil (PTU) is taken daily. The nurse continues to collect data from the client, suspecting that the client has a history of:

1.  Myxedema

2.  Graves' disease

3.  Addison's disease        

4.  Cushing's syndrome     

2.Graves' disease Rationale:

PTU inhibits thyroid hormone synthesis and is used to treat hyperthyroidism, or Graves' disease. Myxedema indicates hypothyroidism.

Cushing's syndrome and Addison's disease are disorders related to adrenal function.

137. 137.) A nurse is reinforcing instructions for a client regarding intranasal desmopressin acetate (DDAVP). The nurse tells the client that which of the following is a side effect of the medication?

1.  Headache

2.  Vulval pain        

3.  Runny nose       

4.  Flushed skin      

3.Runny nose Rationale:

Desmopressin administered by the intranasal route can cause a runny or stuffy nose. Headache, vulval pain, and flushed skin are side effects if the medication is administered by the intravenous (IV) route.

  1. Early morning 138. 138.) A daily dose of prednisone is prescribed for a client.

Rationale: A nurse reinforces instructions to the client regarding

Corticosteroids (glucocorticoids) should be administered before 9:00 administration of the medication and instructs the client

  1. Administration at this time helps minimize adrenal insufficiency that the best time to take this medication is:
  2. At noon and mimics the burst of glucocorticoids released naturally by the
  3. At bedtime adrenal glands each morning.
  4. Early morning *Note the suffix "-sone," and recall that medication names that end
  5. Anytime, at the same time, each day with these letters are corticosteroids.*
  6. An increased amount of daily Humulin NPH insulin
  1. ) Prednisone is prescribed for a client with diabetes

Rationale:

mellitus who is taking Humulin neutral protamine

Glucocorticoids can elevate blood glucose levels. Clients with

Hagedorn (NPH) insulin daily. Which of the following diabetes mellitus may need their dosages of insulin or oral

prescription changes does the nurse anticipate during hypoglycemic medications increased during glucocorticoid therapy. therapy with the prednisone?

  1. An additional dose of prednisone daily Therefore the other options are incorrect.
  2. A decreased amount of daily Humulin NPH insulin
  3. An increased amount of daily Humulin NPH insulin
  4. The addition of an oral hypoglycemic medication daily
  1. Vomiting following cancer chemotherapy
  1. ) The client has a new prescription for

Rationale: metoclopramide (Reglan). On review of the chart, the

Metoclopramide is a gastrointestinal (GI) stimulant andantiemetic. nurse identifies that this medication can be safely

Because it is a GI stimulant, it is contraindicated with GI obstruction, administered with which condition?

  1. Intestinal obstruction hemorrhage, or perforation. It is used in the treatment of emesis after
  2. Peptic ulcer with melena surgery, chemotherapy, and radiation.
  3. Diverticulitis with perforation  
  4. Vomiting following cancer chemotherapy  
  5. "This medication should only be taken with water."
  6. 141.) The nurse has reinforced instructions to a client

Rationale: who has been prescribed cholestyramine (Questran).

Cholestyramine (Questran) is a bile acid sequestrant used to lower the

Which statement by the client indicates a need for cholesterol level, and client compliance is a problem because of its taste further instructions?

  1. "I will continue taking vitamin supplements." and palatability. The use of flavored products or fruit juices can improve 2. "This medication will help lower my cholesterol." the taste. Some side effects of bile acid sequestrants include constipation
  2. "This medication should only be taken with water." and decreased vitamin absorption.
  3. "A high-fiber diet is important while taking this *Note the closed-ended word "only" in option 3* "         

142. 142.) A health care provider has written a prescription for ranitidine (Zantac), once daily. The nurse should schedule the medication for which of the following times? 1. At bedtime

2.  After lunch

3.  With supper       

4.  Before breakfast             

1. At bedtime Rationale:

A single daily dose of ranitidine is usually scheduled to be given at bedtime. This allows for a prolonged effect, and the greatest protection of the gastric mucosa.

*recall that ranitidine suppresses secretions of gastric acids*

143. 143.) A client has just taken a dose of trimethobenzamide (Tigan). The nurse plans to monitor this client for relief of:

1.  Heartburn

2.  Constipation     

3.  Abdominal pain             

4.  Nausea and vomiting             

4. Nausea and vomiting Rationale:

Trimethobenzamide is an antiemetic agent used in the treatment of nausea and vomiting. The other options are incorrect.

  1. Regular bowel movements
  2. ) A client is taking docusate sodium (Colace). The

Rationale: nurse monitors which of the following to determine

Docusate sodium is a stool softener that promotes the absorption of water whether the client is having a therapeutic effect from into the stool, producing a softer consistency of stool. The intended effect this medication?

  1. Abdominal pain is relief or prevention of constipation. The medication does not relieve
  2. Reduction in steatorrhea abdominal pain, stop gastrointestinal (GI) bleeding, or decrease the
  3. Hematest-negative stools amount of fat in the stools.
  4. Regular bowel movements  
  5. Albuterol first and then the beclomethasone dipropionate
  1. ) A nurse has a prescription to give a client

Rationale:

albuterol (Proventil HFA) (two puffs) and

Albuterol is a bronchodilator. Beclomethasone dipropionate is a beclomethasone dipropionate (Qvar) (nasal glucocorticoid. Bronchodilators are always administered before

inhalation, two puffs), by metered-dose inhaler. The glucocorticoids when both are to be given on the same time schedule. nurse administers the medication by giving the:

  1. Albuterol first and then the beclomethasone This allows for widening of the air passages by the bronchodilator, which then makes the glucocorticoid more effective.

dipropionate

  1. Beclomethasone dipropionate first and then the albuterol
  2. Alternating a single puff of each, beginning with the albuterol
  3. Alternating a single puff of each, beginning with the beclomethasone dipropionate
  1. ) A client has begun therapy with theophylline (Theo-24). The nurse tells the client to limit the intake of which of the following while taking this medication? 1. Oranges and pineapple
    1. Coffee, cola, and chocolate
    2. Oysters, lobster, and shrimp Cottage cheese, cream cheese, and dairy creamers
  2. ) A client with a prescription to take theophylline (Theo-24) daily has been given medication instructions by the nurse. The nurse determines that the client needs further information about the medication if the client states that he or she will:
    1. Drink at least 2 L of fluid per day.
    2. Take the daily dose at bedtime. Avoid changing brands of the medication without health care provider (HCP) approval.

  1. Avoid over-the-counter (OTC) cough and cold medications unless approved by the HCP.
  1. ) A client is taking cetirizine hydrochloride (Zyrtec). The nurse checks for which of the following side effects of this medication?
    1. Diarrhea
    2. Excitability
    3. Drowsiness
    4. Excess salivation
  2. ) A client taking fexofenadine (Allegra) is scheduled for allergy skin testing and tells the nurse in the health care provider's office that a dose was taken this morning. The nurse determines that:
    1. The client should reschedule the appointment.
    2. A lower dose of allergen will need to be injected.
    3. A higher dose of allergen will need to be injected.
    4. The client should have the skin test read a day later than usual.
    5. Coffee, cola, and chocolate Rationale:

Theophylline is a xanthine bronchodilator. The nurse teaches the client to limit the intake of xanthine-containing foods while taking this medication. These include coffee, cola, and chocolate.

  1. Take the daily dose at bedtime.

Rationale:

The client taking a single daily dose of theophylline, a xanthine bronchodilator, should take the medication early in the morning. This enables the client to have maximal benefit from the medication during daytime activities. In addition, this medication causes insomnia. The client should take in at least 2 L of fluid per day to decrease viscosity of secretions. The client should check with the physician before changing brands of the medication. The client also checks with the HCP before taking OTC cough, cold, or other respiratory preparations because they could cause interactive effects, increasing the side effects of theophylline and causing dysrhythmias.

  1. Drowsiness Rationale:

A frequent side effect of cetirizine hydrochloride (Zyrtec), an antihistamine, is drowsiness or sedation. Others include blurred vision, hypertension (and sometimes hypotension), dry mouth, constipation, urinary retention, and sweating.

  1. The client should reschedule the appointment.

Rationale:

Fexofenadine is an antihistamine, which provides relief of symptoms caused by allergy. Antihistamines should be discontinued for at least 3 days (72 hours) before allergy skin testing to avoid false-negative readings. This client should have the appointment rescheduled for 3 days after discontinuing the medication.

  1. Double vision, loss of appetite, and nausea
  2. 150.) A client complaining of not feeling well is seen

Rationale: in a clinic. The client is taking several medications

Double vision, loss of appetite, and nausea are signs ofdigoxin toxicity. for the control of heart disease and hypertension.

Additional signs of digoxin toxicity include bradycardia, difficulty reading,

These medications include a β-blocker, digoxin visual alterations such as green and yellow vision or seeing spots or halos,

(Lanoxin), and a diuretic. A tentative diagnosis of confusion, vomiting, diarrhea, decreased libido, and impotence. digoxin toxicity is made. Which of the following

*gastrointestinal (GI) and visual disturbances occur with digoxin toxicity* assessment data would support this diagnosis?

  1. Dyspnea, edema, and palpitations  
  2. Chest pain, hypotension, and paresthesia  
  3. Double vision, loss of appetite, and nausea  
  4. Constipation, dry mouth, and sleep disorder
  5. Monitoring blood pressure
  6. ) A client is being treated for acute congestive

Rationale: heart failure with intravenously administered

Bumetanide is a loop diuretic. Hypotension is a common side effect bumetanide. The vital signs are as follows: blood associated with the use of this medication. The other options also require pressure, 100/60 mm Hg; pulse, 96 beats/min; and assessment but are not the priority. respirations, 24 breaths/min. After the initial dose,

*priority ABCs—airway, breathing, and circulation* which of the following is the priority assessment?

  1. Monitoring weight loss  
  2. Monitoring temperature  
  3. Monitoring blood pressure  
  4. Monitoring potassium level
  5. Protamine sulfate
  1. ) Intravenous heparin therapy is prescribed for a

Rationale: client. While implementing this prescription, a nurse

The antidote to heparin is protamine sulfate; it should be readily available ensures that which of the following medications is for use if excessive bleeding or hemorrhage occurs. Potassium chloride is available on the nursing unit?

  1. Protamine sulfate administered for a potassium deficit. Vitamin K is an antidote for warfarin
  2. Potassium chloride Aminocaproic acid is the antidote for thrombolytic therapy.
  3. Phytonadione (vitamin K )  
  4. Aminocaproic acid (Amicar)  
  5. Blood pressure of 198/110 mm Hg
  1. ) A client is diagnosed with pulmonary embolism

Rationale: and is to be treated with streptokinase (Streptase). A

Thrombolytic therapy is contraindicated in a number of preexisting nurse would report which priority data collection conditions in which there is a risk of uncontrolled bleeding, similar to the finding to the registered nurse before initiating this case in anticoagulant therapy. Thrombolytic therapy also is contraindicated therapy?

  1. Adventitious breath sounds in severe uncontrolled hypertension because of the risk of cerebral
  2. Temperature of 99.4° F orally Therefore the nurse would report the results of the blood
  3. Blood pressure of 198/110 mm Hg pressure to the registered nurse before initiating therapy. The findings in
  4. Respiratory rate of 28 breaths/min options 1, 2, and 4 may be present in the client with pulmonary embolism.

154. 154.) A nurse is reinforcing dietary instructions to a client who has been prescribed cyclosporine (Sandimmune). Which food item would the nurse instruct the client to avoid?

1.  Red meats

2.  Orange juice

3.  Grapefruit juice

4.  Green, leafy vegetables

3.Grapefruit juice Rationale:

A compound present in grapefruit juice inhibits metabolism of cyclosporine. As a result, the consumption of grapefruit juice can raise cyclosporine levels by 50% to 100%, thereby greatly increasing the risk of toxicity. Grapefruit juice needs to be avoided. Red meats, orange juice, and green leafy vegetables are acceptable to consume.

  1. ) Mycophenolate mofetil (CellCept) is prescribed for a client as prophylaxis for organ rejection following an allogeneic renal transplant. Which of the following instructions does the nurse reinforce regarding administration  of this medication? 1. Administer following meals. 2. Take the medication with a magnesium-type antacid.
    1. Open the capsule and mix with food for administration.
    2. Contact the health care provider (HCP) if a sore throat occurs.
  2. ) A nurse is reviewing the laboratory results for a client receiving tacrolimus (Prograf). Which laboratory result would indicate to the nurse that the client is experiencing an adverse effect of the medication? 1. Blood glucose of 200 mg/dL
    1. Potassium level of 3.8 mEq/L
    2. Platelet count of 300,000 cells/mm3 White blood cell count of 6000 cells/mm3
  3. ) A client receiving nitrofurantoin (Macrodantin) calls the health care provider's office complaining of side effects related to the medication. Which side effect indicates the need to stop treatment with this medication?
    1. Nausea
    2. Diarrhea
    3. Anorexia
    4. Cough and chest pain
  4. ) A client with chronic renal failure is receiving epoetin alfa (Epogen, Procrit). Which laboratory result would indicate a therapeutic effect of the medication? 1. Hematocrit of 32%
    1. Platelet count of 400,000 cells/mm3 White blood cell count of 6000 cells/mm3
    2. Blood urea nitrogen (BUN) level of 15 mg/dL
  1. Contact the healthcare provider (HCP) ifa sore throat occurs.

Rationale:

Mycophenolate mofetil should be administered on an empty stomach. The capsules should not be opened or crushed. The client should contact the HCP if unusual bleeding or bruising, sore throat, mouth sores, abdominal pain, or fever occurs because these are adverse effects of the medication. Antacids containing magnesium and aluminum may decrease the absorption of the medication and therefore should not be taken with the medication. The medication may be given in combination with corticosteroids and cyclosporine.

*neutropenia can occur with this medication*

  1. Blood glucose of 200 mg/dL Rationale:

A blood glucose level of 200 mg/dL is elevated above the normal range of 70 to 110 mg/dL and suggests an adverse effect. Other adverse effects include neurotoxicity evidenced by headache, tremor, insomnia; gastrointestinal (GI) effects such as diarrhea, nausea, and vomiting; hypertension; and hyperkalemia.

  1. Cough and chest pain Rationale:

Gastrointestinal (GI) effects are the most frequent adverse reactions to this medication and can be minimized by administering the medication with milk or meals. Pulmonary reactions, manifested as dyspnea, chest pain, chills, fever, cough, and the presence of alveolar infiltrates on the x-ray, would indicate the need to stop the treatment. These symptoms resolve in 2 to 4 days following discontinuation of this medication. *Eliminate options 1, 2, and 3 because they are similar GI-related side effects. Also, use the ABCs— airway, breathing, and circulation*

1.Hematocrit of 32% Rationale:

Epoetin alfa is used to reverse anemia associated with chronic renal failure. A therapeutic effect is seen when the hematocrit is between 30% and 33%. The laboratory tests noted in the other options are unrelated to the use of this medication.

  1. Encouragethe client to cough anddeep breathe.
  1. ) A nurse is caring for a client receiving

Rationale: morphine sulfate subcutaneously for pain.

Morphine sulfate suppresses the cough reflex. Clients need tobe encouraged to

Because morphine sulfate has been prescribed cough and deep breathe to prevent pneumonia. for this client, which nursing action would be

*ABCs—airway, breathing, and circulation* included in the plan of care?

  1. Encourage fluid intake.  
  2. Monitor the client's temperature.  
  3. Maintain the client in a supine position.   Encourage the client to cough and deep      breathe.               
  4. Urinary retention
  1. ) Meperidine hydrochloride (Demerol) is

Rationale: prescribed for the client with pain. Which of the

Meperidine hydrochloride (Demerol) is an opioid analgesic. Side effects of this following would the nurse monitor for as a side medication include respiratory depression, orthostatic hypotension, tachycardia, effect of this medication?

  1. Diarrhea drowsiness and mental clouding, constipation, and urinary retention.
  2. Bradycardia  
  3. Hypertension  
  4. Urinary retention  
  5. Monitor bowel activity.
  1. ) A nurse is caring for a client with severe

Rationale: back pain, and codeine sulfate has been

While the client is taking codeine sulfate, an opioid analgesic, the nurse would prescribed for the client. Which of the monitor vital signs and monitor for hypotension. The nurse should also increase following would the nurse include in the plan fluid intake, palpate the bladder for urinary retention, auscultate bowel sounds, of care while the client is taking this and monitor the pattern of daily bowel activity and stool consistency (codeine medication?

  1. Restrict fluid intake. can cause constipation). The nurse should monitor respiratory status and initiate
  2. Monitor bowel activity. breathing and coughing exercises. In addition, the nurse monitors the
  3. Monitor for hypertension. effectiveness of the pain medication.
  4. Monitor peripheral pulses.  
  5. Liver disease
  1. ) Carbamazepine (Tegretol) is prescribed for

Rationale: a client with a diagnosis of psychomotor

Carbamazepine (Tegretol) is contraindicated in liver disease, and liver function seizures. The nurse reviews the client's health tests are routinely prescribed for baseline purposes and are monitored during history, knowing that this medication is therapy. It is also contraindicated if the client has a history of blood dyscrasias. It contraindicated if which of the following is not contraindicated in the conditions noted in the incorrect options. disorders is present?

  1. Headaches  
  2. Liver disease  
  3. Hypothyroidism  
  4. Diabetes mellitus  
  5. A direct bilirubin level of 2 mg/dL
  1. ) A client with trigeminal neuralgia tells the

Rationale: nurse that acetaminophen (Tylenol) is taken on

In adults, overdose of acetaminophen (Tylenol) causes liver damage. Option 4 is a frequent daily basis for relief of generalized an indicator of liver function and is the only option that indicates an abnormal discomfort. The nurse reviews the client's laboratory value. The normal direct bilirubin is 0 to 0.4 mg/dL. The normal

laboratory results and determines that which of platelet count is 150,000 to 400,000 cells/mm3. The normal prothrombin time is the following indicates toxicity associated with

10 to 13 seconds. The normal sodium level is 135 to 145 mEq/L. the medication?

  1. Sodium of 140 mEq/L
  2. Prothrombin time of 12 seconds
  3. Platelet count of 400,000 cells/mm3
  4. A direct bilirubin level of 2 mg/dL
  1. ) A client receives a prescription for methocarbamol (Robaxin), and the nurse reinforces instructions to the client regarding the medication. Which client statement would indicate a need for further instructions?
  2. "My urine may turn brown or green."
  3. "This medication is prescribed to help relieve my muscle spasms." "If my vision becomes blurred, I don't need to be concerned about

it."

  1. "I need to call my doctor if I experience nasal congestion from this medication."
  2. ) The client has been on treatment for rheumatoid arthritis for 3 weeks. During the administration of etanercept (Enbrel), it is most important for the nurse to assess:
    1. The injection site for itching and

edema

  1. The white blood cell counts and platelet counts
  2. Whether the client is experiencing fatigue and joint pain A metallic taste in the mouth and a loss of appetite
  1. ) Alendronate (Fosamax) is prescribed for a client with osteoporosis. The client taking this medication is instructed to: 1. Take the medication at bedtime. 2. Take the medication in the morning with breakfast.
    1. Lie down for 30 minutes after taking the medication.
    2. Take the medication with a full glass of water after rising in the morning.
    3. "If my vision becomesblurred, I don't need tobe concerned about it." Rationale:

The client needs to be told that the urine may turn brown, black, or green. Other adverse effects include blurred vision, nasal congestion, urticaria, and rash. The client needs to be instructed that, if these adverse effects occur, the health care provider needs to be notified. The medication is used to relieve muscle spasms.

  1. The white blood cell counts and platelet counts Rationale:

Infection and pancytopenia are adverse effects of etanercept (Enbrel). Laboratory studies are performed before and during treatment. The appearance of abnormal white blood cell counts and abnormal platelet counts can alert the nurse to a potential life-threatening infection. Injection site itching is a common occurrence following administration of the medication. In early treatment, residual fatigue and joint pain may still be apparent. A metallic taste and loss of appetite are not common signs of side effects of this medication.

4.Take the medication with a full glass of water after rising in the morning. Rationale:

Precautions need to be taken with administration of alendronate to prevent gastrointestinal side effects (especially esophageal irritation) and to increase absorption of the medication. The medication needs to be taken with a full glass of water after rising in the morning. The client should not eat or drink anything for 30 minutes following administration and should not lie down after taking the medication.

  1. Instruct the client todrink 3000 mL of fluidper day.
  1. ) A nurse prepares to reinforce

Rationale: instructions to a client who is taking

Allopurinol (Zyloprim) is an antigout medication used to decrease uric acid levels. allopurinol (Zyloprim). The nurse plans

Clients taking allopurinol are encouraged to drink 3000 mL of fluid a day. A full to include which of the following in the therapeutic effect may take 1 week or longer. Allopurinol is to be given with or instructions?

  1. Instruct the client to drink 3000 mL immediately following meals or milk to prevent gastrointestinal irritation. If the client develops a rash, irritation of the eyes, or swelling of the lips or mouth, he or she should of fluid per day.
  2. Instruct the client to take the contact the health care provider because this may indicate hypersensitivity.

        medication on an empty stomach.           

  1. Inform the client that the effect of the medication will occur immediately.   Instruct the client that, if swelling of  the lips occurs, this is a normal  expected response.  
  2. Renal failure
  1. ) Colcrys (colchicine) is prescribed

Rationale: for a client with a diagnosis of gout.

Colchicine is contraindicated in clients with severe gastrointestinal, renal, hepatic or

The nurse reviews the client's medical cardiac disorders, or with blood dyscrasias. Clients with impaired renal function may history in the health record, knowing exhibit myopathy and neuropathy manifested as generalized weakness. This medication that the medication would be should be used with caution in clients with impaired hepatic function, older clients, and contraindicated in which disorder?

  1. Myxedema debilitated clients.
  2. Renal failure *Note that options 1, 3, and 4 are all endocrine-related disorders:
  3. Hypothyroidism Myxedema=Hypothyroidism*
  4. Diabetes mellitus  
  5. Once daily, at the same time each day
  1. ) Insulin glargine (Lantus) is

Rationale: prescribed for a client with diabetes

Insulin glargine is a long-acting recombinant DNA human insulin used to treat type 1 and mellitus. The nurse tells the client that  type 2 diabetes mellitus. It has a 24-hour duration of action and is administered once a it is best to take the insulin:

  1. 1 hour after each meal day, at the same time each day.
  2. Once daily, at the same time each day
  3. 15 minutes before breakfast, lunch, and dinner
  4. Before each meal, on the basis of the blood glucose level
  1. Check the client's bloodpressure.
  1. ) Atenolol hydrochloride

Rationale:

(Tenormin) is prescribed for a

Atenolol hydrochloride is a beta-blocker used to treat hypertension. Therefore the priority hospitalized client. The nurse nursing action before administration of the medication is to check the client's blood pressure. should perform which of the

The nurse also checks the client's apical heart rate. If the systolic blood pressure is below 90 following as a priority action mm Hg or the apical pulse is 60 beats per minute or lower, the medication is withheld and before administering the the registered nurse and/or health care provider is notified. The nurse would check baseline medication?

  1. Listen to the client's lung sounds. renal and liver function tests. The medication may cause weakness, and the nurse would
  2. Check the client's blood assist the client with activities if weakness occurs.

*Beta-blockers have "-lol" at the end of the medication name* pressure.

  1. Check the recent electrolyte      
  2. Assess the client for muscle       

171. 171.) A nurse is preparing to administer furosemide (Lasix) to a client with a diagnosis of heart failure. The most important laboratory test result for the nurse to check before administering this medication is:                

1.  Potassium level             

2.  Creatinine level             

3.  Cholesterol level             

4.  Blood urea nitrogen             

1.Potassium level Rationale:

Furosemide is a loop diuretic. The medication causes a decrease in the client's electrolytes, especially potassium, sodium, and chloride. Administering furosemide to a client with low electrolyte levels could precipitate ventricular dysrhythmias. Options 2 and 4 reflect renal function. The cholesterol level is unrelated to the administration of this medication.

172. 172.) A nurse provides dietary instructions to a client who will be taking warfarin sodium (Coumadin). The nurse tells the client to avoid which food item?

1.  Grapes

2.  Spinach             

3.  Watermelon      

4.  Cottage cheese             

2.Spinach Rationale:

Warfarin sodium is an anticoagulant. Anticoagulant medications act by antagonizing the action of vitamin K, which is needed for clotting. When a client is taking an anticoagulant, foods high in vitamin K often are omitted from the diet. Vitamin K-rich foods include green, leafy vegetables, fish, liver, coffee, and tea.

  1. "Do you have any joint pain?"
  2. ) A nurse reviews the

Rationale: medication history of a client

Leflunomide is an immunosuppressive agent and has an anti-inflammatory action. The admitted to the hospital and notes medication provides symptomatic relief of rheumatoid arthritis. Diarrhea can occur as a side that the client is taking leflunomide effect of the medication. The other options are unrelated to medication effectiveness.

(Arava). During data collection, the nurse asks which question to determine medication effectiveness?

  1. "Do you have any joint pain?"
  2. "Are you having any diarrhea?" "Do you have frequent headaches?"
  3. "Are you experiencing heartburn?"
  1. Blood ammonialevel
  1. ) A client with portosystemic

Rationale: encephalopathy is receiving oral lactulose

Lactulose is a hyperosmotic laxative and ammonia detoxicant. It is used to prevent or

(Chronulac) daily. The nurse assesses which treat portosystemic encephalopathy, including hepatic precoma and coma. It also is of the following to determine medication used to treat constipation. The medication retains ammonia in the colon (decreases effectiveness?

  1. Lung sounds the blood ammonia concentration), producing an osmotic effect. It promotes
  2. Blood pressure increased peristalsis and bowel evacuation, expelling ammonia from the colon.
  3. Blood ammonia level  
  4. Serum potassium level  
    1. Human immunodeficiency virus (HIV) infection
  5. ) A nurse notes that a client is

Rationale: receiving lamivudine (Epivir). The nurse

Lamivudine is a nucleoside reverse transcriptase inhibitor and antiviral medication. It determines that this medication has been slows HIV replication and reduces the progression of HIV infection. It also is used to prescribed to treat which of the following?

  1. Pancreatitis treat chronic hepatitis B and is used for prophylaxis in health care workers at risk of
  2. Pharyngitis acquiring HIV after occupational exposure to the virus.
  3. Tonic-clonic seizures *Note the letters "-vir" in the trade name for this medication *
  4. Human immunodeficiency virus (HIV)  infection               
  5. "Are you experiencing any heartburn?"
  1. ) A nurse notes that a client is taking

Rationale: lansoprazole (Prevacid). On data

Lansoprazole is a gastric acid pump inhibitor used to treat gastric and duodenal collection, the nurse asks which question ulcers, erosive esophagitis, and hypersecretory conditions. It also is used to treat to determine medication effectiveness?

  1. "Has your appetite increased?" gastroesophageal reflux disease (GERD). It is not used to treat visual problems,
  2. "Are you experiencing any heartburn?" problems with appetite, or leg pain.
  3. "Do you have any problems with vision?" *NOTE: "-zole" refers to gastric acid pump inhibitors*
  4. "Do you experience any leg pain when walking?"       
    1. The client is experiencing magnesium toxicity.
  5. ) A nurse is assisting in caring for a

Rationale: pregnant client who is receiving

Magnesium toxicity can occur as a result of magnesium sulfate therapy. Signs of intravenous magnesium sulfate for the magnesium sulfate toxicity relate to the central nervous system depressant effects of management of preeclampsia and notes the medication and include respiratory depression; loss of deep tendon reflexes; that the client's deep tendon reflexes are sudden decrease in fetal heart rate or maternal heart rate, or both; and sudden drop absent. On the basis of this data, the nurse in blood pressure. Hyperreflexia indicates increased cerebral edema. An absence of reports the finding and makes which reflexes indicates magnesium toxicity. The therapeutic serum level of magnesium for a determination?

  1. The magnesium sulfate is effective. client receiving magnesium sulfate ranges from 4 to 7.5 mEq/L (5 to 8 mg/dL).
  2. The infusion rate needs to be increased.
  3. The client is experiencing cerebral edema.
  4. The client is experiencing magnesium toxicity.
  1. ) Methylergonovine (Methergine) is prescribed for a client with postpartum hemorrhage caused by uterine atony. Before administering the medication, the nurse checks which of the following as the important client parameter? 1. Temperature
    1. Lochial flow
    2. Urine output
    3. Blood pressure
  2. ) A nurse provides medication instructions to a client who had a kidney transplant about therapy with cyclosporine (Sandimmune). Which statement by the client indicates a need for further instruction?
    1. "I need to obtain a yearly influenza vaccine." "I need to have dental checkups every 3 months." 3. "I need to self-monitor my blood pressure at home." 4. "I need to call the health care provider (HCP) if my urine volume decreases or my urine becomes cloudy."
  3. ) A health care provider (HCP) writes a prescription for digoxin (Lanoxin), 0.25 mg daily. The nurse teaches the client about the medication and tells the client that it is important to: 1. Count the radial and carotid pulses every morning.
    1. Check the blood pressure every morning and evening.
    2. Stop taking the medication if the pulse is higher than 100 beats per minute.
    3. Withhold the medication and call the HCP if the pulse is less than 60 beats per minute.
  1. Blood pressure Rationale:

Methylergonovine is an ergot alkaloid used for postpartum hemorrhage. It stimulates contraction of the uterus and causes arterial vasoconstriction. Ergot alkaloids are avoided in clients with significant cardiovascular disease, peripheral disease, hypertension, eclampsia, or preeclampsia. These conditions are worsened by the vasoconstrictive effects of the ergot alkaloids. The nurse would check the client's blood pressure before administering the medication and would follow agency protocols regarding withholding of the medication.Options 1, 2, and 3 are items that are checked in the postpartum period, but they are unrelated to the use of this medication.

  1. "I need to obtain a yearly influenza vaccine." Rationale:

Cyclosporine is an immunosuppressant medication. Because of the medication's effects, the client should not receive any vaccinations without first consulting the HCP. The client should report decreased urine output or cloudy urine, which could indicate kidney rejection or infection, respectively. The client must be able to self-monitor blood pressure to check for the side effect of hypertension. The client needs meticulous oral care and dental cleaning every 3 months to help prevent gingival hyperplasia.

4.Withhold the medication and call the HCP if the pulse is less than 60 beats per minute. Rationale:

An important component of taking this medication is monitoring the pulse rate; however, it is not necessary for the client to take both the radial and carotid pulses. It is not necessary for the client to check the blood pressure every morning and evening because the medication does not directly affect blood pressure. It is most important for the client to know the guidelines related to withholding the medication and calling the HCP. The client should not stop taking a medication.

  1. Acetylsalicylic acid (aspirin)
  1. ) A client is taking ticlopidine hydrochloride (Ticlid). The

Rationale: nurse tells the client to avoid which of the following while

Ticlopidine hydrochloride is a platelet aggregation inhibitor. It is taking this medication?

  1. Vitamin C used to decrease the risk of thrombotic strokes in clients with
  2. Vitamin D precursor symptoms. Because it is an antiplatelet agent, other
  3. Acetaminophen (Tylenol) medications that precipitate or aggravate bleeding should be
  4. Acetylsalicylic acid (aspirin)avoided during its use. Therefore, aspirin or any aspirin-containing product should be avoided.

Rationale:

  1. ) A client with angina pectoris is experiencing chest

Nitroglycerin acts directly on the smooth muscle of the blood pain that radiates down the left arm. The nurse administers vessels, causing relaxation and dilation. As a result, hypotension can a sublingual nitroglycerin tablet to the client. The client's occur. The nurse would check the client's blood pressure before pain is unrelieved, and the nurse determines that the client administering the second nitroglycerin tablet. Although the needs another nitroglycerin tablet. Which of the following respirations and apical pulse may be checked, these vital signs are vital signs is most important for the nurse to check before not affected as a result of this medication. The temperature also is administering the medication?

  1. Temperature not associated with the administration of this medication.
  2. Respirations  
  3. Blood pressure  
  4. Radial pulse rate  
  5. "I need to discontinue the medication after 14 days of use."
  1. ) A client who received a kidney transplant is taking

Rationale: azathioprine (Imuran), and the nurse provides instructions

Azathioprine is an immunosuppressant medication that is taken for about the medication. Which statement by the client life. Because of the effects of the medication, the client must watch indicates a need for further instructions?

  1. "I need to watch for signs of infection." for signs of infection, which are reported immediately to the HCP.
  2. "I need to discontinue the medication after 14 days of The client should also call the HCP if more than one dose is missed.

The medication may be taken with meals to minimize nausea. use."

  1. "I can take the medication with meals to minimize  "              
  2. "I need to call the health care provider (HCP) if more than one dose is missed."  
  3. Prednisone
  1. ) A nurse preparing a client for surgery reviews the

Rationale: client's medication record. The client is to be nothing per

Prednisone is a corticosteroid that can cause adrenal atrophy, which mouth (NPO) after midnight. Which of the following reduces the body's ability to withstand stress. Before and during medications, if noted on the client's record, should the surgery, dosages may be temporarily increased. Cyclobenzaprine is nurse question?

  1. Cyclobenzaprine (Flexeril) a skeletal muscle relaxant. Alendronate is a bone-resorption
  2. Alendronate (Fosamax) Allopurinol is an antigout medication.
  3. Allopurinol (Zyloprim)  
  4. Prednisone  

185. 185.) Which of the following herbal therapies would be prescribed for its use as an antispasmodic? Select all that apply.

1.Aloe

2.Kava

3.Ginger

4.Chamomile

5.Peppermint oil

4.Chamomile 5.Peppermint oil Rationale:

Chamomile has a mild sedative effect and acts as an antispasmodic and anti-inflammatory. Peppermint oil acts as an antispasmodic and is used for irritable bowel syndrome. Topical aloe promotes wound healing. Aloe taken orally acts as a laxative. Kava has an anxiolytic, sedative, and analgesic effect. Ginger is effective in relieving nausea.

  1. ) A nurse prepares to administer sodium polystyrene sulfonate (Kayexalate) to a client. Before administering the medication, the nurse reviews the action of the medication and understands that it:
  • Releases bicarbonate in exchange for primarily sodium ions Releases sodium ions in exchange for primarily potassium ions 3. Releases potassium ions in exchange for primarily sodium ions 4. Releases sodium ions in exchange for primarily bicarbonate ions
  1. ) A clinic nurse prepares to administer an MMR (measles, mumps, rubella) vaccine to a child. How is this vaccine best administered?
    1. Intramuscularly in the deltoid muscle
    2. Subcutaneously in the gluteal muscle
  • Subcutaneously in the outer aspect of the upper arm Intramuscularly in the anterolateral aspect of the thigh
  1. ) The nurse should anticipate that the most likely medication to be prescribed prophylactically for a child with spina bifida

(myelomeningocele) who has a neurogenic bladder would be:

  1. Prednisone
  2. Sulfisoxazole
  3. Furosemide (Lasix)
  4. Intravenous immune globulin

(IVIG)

  1. ) Prostaglandin E1 is prescribed for a child with transposition of the great arteries. The mother of  the child asks the nurse why the child needs the medication. The nurse tells the mother that the medication:
    1. Prevents hypercyanotic (blue or tet) spells
    2. Maintains an adequate hormone level
    3. Maintains the position of the great arteries
    4. Provides adequate oxygen saturation and maintains cardiac output
  2. Releases sodium ions inexchange for primarily potassium ions Rationale:

Sodium polystyrene sulfonate is a cation exchange resin used in the treatment of hyperkalemia. The resin either passes through the intestine or is retained in the colon. It releases sodium ions in exchange for primarily potassium ions. The therapeutic effect occurs 2 to 12 hours after oral administration and longer after rectal administration.

  1. Subcutaneously in the outer aspect of the upper arm Rationale:

The MMR vaccine is administered subcutaneously in the outer aspect of the upper arm. The gluteal muscle is most often used for intramuscular injections. The MMR vaccine is not administered by the intramuscular route.

  1. Sulfisoxazole Rationale:

A neurogenic bladder prevents the bladder from completely emptying because of the decrease in muscle tone. The most likely medication to be prescribed to prevent urinary tract infection would be an antibiotic. A common prescribed medication is sulfisoxazole. Prednisone relieves allergic reactions and inflammation rather than preventing infection.

Furosemide promotes diuresis and decreases edema caused by congestive heart failure. IVIG assists with antibody production in immunocompromised clients.  

4.Provides adequate oxygen saturation and maintains cardiac output Rationale:

A child with transposition of the great arteries may receive prostaglandin E1 temporarily to increase blood mixing if systemic and pulmonary mixing are inadequate to maintain adequate cardiac output. Options 1, 2, and 3 are incorrect. In addition, hypercyanotic spells occur in tetralogy of Fallot.

*Use the ABCs—airway, breathing, and circulation—to answer the question. The correct option addresses circulation*

  1. Dimercaprol (BAL inOil)
  2. 190.) A child is hospitalized with a

Rationale: diagnosis of lead poisoning. The nurse

Dimercaprol is a chelating agent that is administered to remove lead from the circulating assisting in caring for the child would blood and from some tissues and organs for excretion in the urine. Sodium bicarbonate prepare to assist in administering which may be used in salicylate poisoning. Syrup of ipecac is used in the hospital setting in of the following medications?

  1. Activated charcoal poisonings to induce vomiting. Activated charcoal is used to decrease absorption in 2. Sodium bicarbonate certain poisoning situations. Note that dimercaprol is prepared with peanut oil, and
  2. Syrup of ipecac syrup hence should be avoided by clients with known or suspected peanut allergy. 4. Dimercaprol (BAL in Oil)

191. 191.) A child is brought to the emergency department for treatment of an acute asthma attack. The nurse prepares to administer which of the following medications first?

1.  Oral corticosteroids        

2.  A leukotriene modifier    

3.  A β2 agonist      

4.  A nonsteroidal anti-inflammatory    

3. A β2 agonist Rationale:

In treating an acute asthma attack, a short acting β2 agonist such as albuterol (Proventil HFA) will be given to produce bronchodilation. Options 1, 2, and 4 are long-term control (preventive) medications.

192. 192.) A nurse is collecting medication information from a client, and the client states that she is taking garlic as an herbal supplement. The nurse understands that the client  is  most likely treating which of the following conditions?             

1.  Eczema             

2.  Insomnia           

3.  Migraines          

4.  Hyperlipidemia             

4.Hyperlipidemia Rationale:

Garlic is an herbal supplement that is used to treat hyperlipidemia and hypertension. An herbal supplement that may be used to treat eczema is evening primrose. Insomnia has been treated with both valerian root and chamomile. Migraines have been treated with feverfew.

  1. Ensure that the solution is freshly prepared before use.
  2. ) Sodium hypochlorite (Dakin's

Rationale: solution) is prescribed for a client with a

Dakin solution is a chloride solution that is used for irrigating and cleaning necrotic or leg wound containing purulent purulent wounds. It can be used for packing necrotic wounds. It cannot be used to pack drainage. The nurse is assisting in purulent wounds because the solution is inactivated by copious pus. It should not come developing a plan of care for the client into contact with healing or normal tissue, and it should be rinsed off immediately if and includes which of the following in used for irrigation. Solutions are unstable and the nurse must ensure that the solution has the plan?

  1. Ensure that the solution is freshly been prepared fresh before use.

*Eliminate options 2 and 3 first because they are comparable or alike. It makes sense prepared before use.

  1. Soak a sterile dressing with solution to ensure that the solution is freshly prepared; therefore, select option 1* and pack into the wound.
  2. Allow the solution to remain in the wound following irrigation.
  3. Apply the solution to the wound and on normal skin tissue surrounding the wound.
  1. ) A nurse provides instructions to a client regarding the use of tretinoin (Retin-A). Which statement by the client indicates the need for further instructions?
    1. "Optimal results will be seen after 6 weeks."
    2. "I should apply a very thin layer to my skin."

  • "I should wash my hands thoroughly after applying the medication." "I should cleanse my skin thoroughly before applying the medication."
  1. ) A nurse is caring for a client who is taking metoprolol (Lopressor). The nurse measures the client's blood pressure (BP) and apical pulse (AP) immediately before administration. The client's BP is 122/78 mm/Hg and the AP is 58 beats/min. Based on this data, which of the following is the appropriate action? 1. Withhold the medication.
    1. Notify the registered nurse immediately.
    2. Administer the medication as prescribed.
    3. Administer half of the prescribed medication.
    4. "I should apply a verythin layer to my skin." Rationale:

Tretinoin is applied liberally to the skin. The hands are washed thoroughly immediately after applying. Therapeutic results should be seen after 2 to 3 weeks but may not be optimal until after 6 weeks. The skin needs to be cleansed thoroughly before applying the medication.

  1. Withhold the medication.

Rationale:

Metoprolol (Lopressor) is classified as a beta-adrenergic blocker and is used in the treatment of hypertension, angina, and myocardial infarction. Baseline nursing assessments include measurement of BP and AP immediately before administration. If the systolic BP is below 90 mm/Hg and the AP is below 60 beats/min, the nurse should withhold the medication and document this action. Although the registered nurse should be informed of the client's vital signs, it is not necessary to do so immediately. The medication should not be administered because the data is outside of the prescribed parameters for this medication. The nurse should not administer half of the medication, or alter any dosages at any point in time.

196. 196.) A client has been prescribed amikacin (Amikin). Which of the following priority baseline functions should be monitored?

1.  Apical pulse

2.  Liver function

3.  Blood pressure

4.  Hearing acuity

4.Hearing acuity Rationale:

Amikacin (Amikin) is an antibiotic. This medication can cause ototoxicity and nephrotoxicity; therefore, hearing acuity tests and kidney function studies should be performed before the initiation of therapy. Apical pulse, liver function studies, and blood pressure are not specifically related to the use of this medication.

  • ) Collagenase (Santyl) is prescribed for a client with a severe burn to the hand. The nurse provides instructions to the client regarding the use of the medication. Which statement by the client indicates an accurate understanding of the use of this medication? 1. "I will apply the ointment once a day and leave it open to the air."
  1. "I will apply the ointment twice a day and leave it open to the air." 3. "I will apply the ointment once a day and cover it with a sterile dressing."
  2. "I will apply the ointment at bedtime and in the morning and cover it with a sterile dressing."
  • ) Coal tar has been prescribed for a client with a diagnosis of psoriasis, and the nurse provides instructions to the client about the medication. Which statement by the client indicates a need for further instructions?
    1. "The medication can cause phototoxicity."
    2. "The medication has an unpleasant odor."
    3. "The medication can stain the skin and hair."
    4. "The medication can cause systemic effects."

 

  • ) A nurse is applying a topical glucocorticoid to a client with eczema. The nurse monitors for systemic absorption of the medication if the medication is being applied to which of the following body areas?
    1. Back
    2. Axilla
    3. Soles of the feet
    4. Palms of the hands

 

  • ) A client is seen in the clinic for complaints of skin itchiness that has been persistent over the past several weeks. Following data collection, it has been determined that the client has scabies. Lindane is prescribed, and the nurse is asked to provide instructions to the client regarding the use of the medication. The nurse tells the client to:
    1. Apply a thick layer of cream to the entire body.
    2. Apply the cream as prescribed for 2 days in a row.
    3. Apply to the entire body and scalp, excluding the face.
    4. Leave the cream on for 8 to 12 hours and then remove by washing.
  1. "I will apply the ointment once a day and cover it with a sterile dressing."Rationale:

Collagenase is used to promote debridement of dermal lesions and severe burns. It is usually applied once daily and covered with a sterile dressing.

  1. "The medication can cause systemic effects." Rationale:

Coal tar is used to treat psoriasis and other chronic disorders of the skin. It suppresses DNA synthesis, mitotic activity, and cell proliferation. It has an unpleasant odor, can frequently stain the skin and hair, and can cause phototoxicity. Systemic toxicity does not occur. *The name of the medication will assist in eliminating options 2 and 3*

  1. Axilla

Rationale:

Topical glucocorticoids can be absorbed into the systemic circulation. Absorption is higher from regions where the skin is especially permeable (scalp, axillae, face, eyelids, neck, perineum, genitalia), and lower from regions where penetrability is poor (back, palms, soles). *Eliminate options 3 and 4 because these body areas are similar in terms of skin characteristics*

  1. Leave the cream on for 8 to 12 hours and then remove by washing.

Rationale:

Lindane is applied in a thin layer to the entire body below the head. No more than 30 g (1 oz) should be used. The medication is removed by washing 8 to 12 hours later.

Usually, only one application is required.

201. 201.) A nurse is preparing to administer eardrops to an infant. The nurse plans to:

1.       Pull up and back on the ear and direct the solution onto the eardrum.

2.       Pull down and back on the ear and direct the solution onto the eardrum.

3.       Pull down and back on the ear and direct the solution toward the wall of the canal.

4.       Pull up and back on the ear lobe and direct the solution toward the wall of the canal.

3.Pull down and back on the ear and direct the solution toward the wall of the canal.

Rationale:

When administering eardrops to an infant, the nurse pulls the ear down and straight back. In the adult or a child older than 3 years, the ear is pulled up and back to straighten the auditory canal. The medication is administered by aiming it at the wall of the canal rather than directly onto the eardrum.

  • ) A nurse is collecting data from a client about medications being taken, and the client tells the nurse that he is taking herbal supplements for the treatment of varicose veins. The nurse understands that the client is most likely taking which of the following?
    1. Bilberry
    2. Ginseng
    3. Feverfew
    4. Evening primrose
  • ) A nurse is preparing to give the postcraniotomy client medication for incisional pain. The family asks the nurse why the client is receiving codeine sulfate and not "something stronger." In formulating a response, the nurse incorporates the understanding that codeine: 1. Is one of the strongest opioid analgesics available 2. Cannot lead to physical or psychological dependence 3. Does not cause gastrointestinal upset or constipation as do other opioids

  1. Does not alter respirations or mask neurological signs as do other opioids
  • ) A client receives a dose of edrophonium (Enlon). The client shows improvement in muscle strength for a period of time following the injection. The nurse interprets that this finding is compatible with: 1. Multiple sclerosis
    1. Myasthenia gravis
    2. Muscular dystrophy
    3. Amyotrophic lateral sclerosis
  1. Bilberry Rationale:

Bilberry is an herbal supplement that has been used to treat varicose veins. This supplement has also been used to treat cataracts, retinopathy, diabetes mellitus, and peripheral vascular disease. Ginseng has been used to improve memory performance and decrease blood glucose levels in type 2 diabetes mellitus. Feverfew is used to prevent migraine headaches and to treat rheumatoid arthritis. Evening primrose is used to treat eczema and skin irritation.

  1. Does not alter respirations or mask neurological signs as do other opioids Rationale:

Codeine sulfate is the opioid analgesic often used for clients after craniotomy. It is frequently combined with a nonopioid analgesic such as acetaminophen for added effect. It does not alter the respiratory rate or mask neurological signs as do other opioids. Side effects of codeine include gastrointestinal upset and constipation. The medication can lead to physical and psychological dependence with chronic use. It is not the strongest opioid analgesic available.

2.Myasthenia gravis Rationale:

Myasthenia gravis can often be diagnosed based on clinical signs and symptoms. The diagnosis can be confirmed by injecting the client with a dose of edrophonium . This medication inhibits the breakdown of an enzyme in the neuromuscular junction, so more acetylcholine binds to receptors. If the muscle is strengthened for 3 to 5 minutes after this injection, it confirms a diagnosis of myasthenia gravis. Another medication, neostigmine (Prostigmin), also may be used because its effect lasts for 1 to 2 hours, providing a better analysis. For either medication, atropine sulfate should be available as the antidote.

  • ) A nurse is assisting in preparing to administer acetylcysteine (Mucomyst) to a client with an overdose of acetaminophen (Tylenol). The nurse prepares to administer the medication by:
    1. Administering the medication subcutaneously in the deltoid muscle
    2. Administering the medication by the intramuscular route in the gluteal muscle Administering the medication by the intramuscular route, mixed in 10 mL of normal saline
    3. Mixing the medication in a flavored ice drink and allowing the client to drink the medication through a straw
  • ) A client is receiving baclofen (Lioresal) for muscle spasms caused by a spinal cord injury. The nurse monitors the client, knowing that which of the following is a side effect of this medication? 1. Muscle pain
    1. Hypertension
    2. Slurred speech
    3. Photosensitivity
  • ) A client is suspected of having myasthenia gravis, and the health care provider administers edrophonium (Enlon) to determine the diagnosis. After administration of this medication, which of the following would indicate the presence of myasthenia gravis?
    1. Joint pain
    2. A decrease in muscle strength
    3. An increase in muscle strength
    4. Feelings of faintness, dizziness, hypotension, and signs of flushing in the client
  • ) A client with myasthenia gravis verbalizes complaints of feeling much weaker than normal. The health care provider plans to implement a diagnostic test to determine if the client is experiencing a myasthenic crisis and administers edrophonium (Enlon). Which of the following would indicate that the client is experiencing a myasthenic crisis? 1. Increasing weakness
    1. No change in the condition
    2. An increase in muscle spasms
    3. A temporary improvement in the condition
  1. Mixing the medication in a flavored ice drink and allowing the client to drink the medication through a strawRationale:

Because acetylcysteine has a pervasive odor of rotten eggs, it must be disguised in a flavored ice drink. It is consumed preferably through a straw to minimize contact with the mouth. It is not administered by the intramuscular or subcutaneous route.

*Knowing that the medication is a solution that is also used for nebulization treatments will assist you to select the option that indicates an oral route*

Rationale:

Side effects of baclofen include drowsiness, dizziness, weakness, and nausea. Occasional side effects include headache, paresthesia of the hands and feet, constipation or diarrhea, anorexia, hypotension, confusion, and nasal congestion. Paradoxical central nervous system excitement and restlessness can occur, along with slurred speech, tremor, dry mouth, nocturia, and impotence.

*Option 3 is most closely associated with a neurological disorder*

3.An increase in muscle strength Rationale:

Edrophonium is a short-acting acetylcholinesterase inhibitor used as a diagnostic agent. When a client with suspected myasthenia gravis is given the medication intravenously, an increase in muscle strength would be seen in 1 to 3 minutes. If no response occurs, another dose is given over the next 2 minutes, and muscle strength is again tested. If no increase in muscle strength occurs with this higher dose, the muscle weakness is not caused by myasthenia gravis. Clients receiving injections of this medication commonly demonstrate a drop of blood pressure, feel faint and dizzy, and are flushed.

auto-define "A client with myasthen..." Rationale:

Edrophonium (Enlon) is administered to determine whether the client is reacting to an overdose of a medication (cholinergic crisis) or to an increasing severity of the disease (myasthenic crisis). When the edrophonium (Enlon) injection is given and the condition improves temporarily, the client is in myasthenic crisis. This is known as a positive test. Increasing weakness would occur in cholinergic crisis. Options 2 and 3 would not occur in either crisis.

  1. Drowsiness
  2. 209.) A client with multiple sclerosis is receiving diazepam

Rationale:

(Valium), a centrally acting skeletal muscle relaxant. Which of

Incoordination and drowsiness are common side effects resulting the following would indicate that the client is experiencing a from this medication. Options 1, 3, and 4 are incorrect. side effect related to this medication?

  1. Headache
  2. Drowsiness
  3. Urinary retention  
  4. Increased salivation  

210. 210.) Dantrolene (Dantrium) is prescribed for a client with a spinal cord injury for discomfort resulting from spasticity. The nurse tells the client about the importance of follow-up and the need for which blood study?

1.  Creatinine level

2.  Sedimentation rate

3.  Liver function studies

4.  White blood cell count            

3.Liver function studies Rationale:

Dantrolene can cause liver damage, and the nurse should monitor liver function studies. Baseline liver function studies are done before therapy starts, and regular liver function studies are performed throughout therapy. Dantrolene is discontinued if no relief of spasticity is achieved in 6 weeks.

211. 211.) A client with epilepsy is taking the prescribed dose of phenytoin (Dilantin) to control seizures. A phenytoin blood level is drawn, and the results reveal a level of 35 mcg/ml. Which of the following symptoms would be expected as a result of this laboratory result?

1.  Nystagmus

2.  Tachycardia     

3.  Slurred speech                

4.  No symptoms, because this is a normal therapeutic level  

3.Slurred speech Rationale:

The therapeutic phenytoin level is 10 to 20 mcg/mL. At a level higher than 20 mcg/mL, involuntary movements of the eyeballs (nystagmus) appear. At a level higher than 30 mcg/mL, ataxia and slurred speech occur.

  1. Induces diuresis by raising the osmotic pressure of glomerular
  2. ) Mannitol (Osmitrol) is being administered to a client with filtrate, thereby inhibiting tubular reabsorption of water and increased intracranial pressure following a head injury. The

solutes nurse assisting in caring for the client knows that which of the

Rationale: following indicates the therapeutic action of this medication?

  1. Prevents the filtration of sodium and water through the Mannitol is an osmotic diuretic that induces diuresis by raising the osmotic pressure of glomerular filtrate, thereby inhibiting tubular kidneys
  2. Prevents the filtration of sodium and potassium through the reabsorption of water and solutes. It is used to reduce intracranial pressure in the client with head trauma. kidneys
  3. Decreases water loss by promoting the reabsorption of sodium and water in the loop of Henle
  4. Induces diuresis by raising the osmotic pressure of glomerular filtrate, thereby inhibiting tubular reabsorption of water and solutes
  1. ) A client is admitted to the hospital with complaints of back spasms. The client states, "I have been taking two or three aspirin every 4 hours for the  past  week and it hasn't helped my back." Aspirin intoxication is suspected. Which of the following complaints would indicate aspirin intoxication? 1. Tinnitus
    1. Constipation
    2. Photosensitivity
    3. Abdominal cramps
  2. ) A health care provider initiates carbidopa/levodopa (Sinemet) therapy for the client with Parkinson's disease. A few days after the client starts the medication, the client complains of nausea and vomiting.

The nurse tells the client that:

  1. Taking an antiemetic is the best measure to prevent the nausea.
  2. Taking the medication with food will help to prevent the nausea.
  3. This is an expected side effect of the medication and will decrease over time.
  4. The nausea and vomiting will decrease when the dose of levodopa is stabilized.
  1. ) A client with rheumatoid arthritis is taking acetylsalicylic acid (aspirin) on a daily basis. Which medication dose should the nurse expect the client to be taking?
    1. 1 g daily
    2. 4 g daily
    3. 325 mg daily
    4. 1000 mg daily

 

 

  1. ) A nurse is caring for a client with gout who is taking Colcrys (colchicine). The client has been instructed to restrict the diet to low-purine foods. Which of the following foods should the nurse instruct the client to avoid while taking this medication?
    1. Spinach
    2. Scallops
    3. Potatoes
    4. Ice cream
  1. Tinnitus

Rationale:

Mild intoxication with acetylsalicylic acid (aspirin) is called salicylism and is commonly experienced when the daily dosage is higher than 4 g. Tinnitus (ringing in the ears) is the most frequently occurring effect noted with intoxication. Hyperventilation may occur because salicylate stimulates the respiratory center. Fever may result because salicylate interferes with the metabolic pathways involved with oxygen consumption and heat production. Options [18], 3, and 4 are incorrect.  

  1. Taking the medication with food will help to prevent the nausea. Rationale:

If carbidopa/levodopa is causing nausea and vomiting, the nurse would tell the client that taking the medication with food will prevent the nausea. Additionally, the client should be instructed not to take the medication with a high-protein meal because the high-protein will affect absorption. Antiemetics from the phenothiazine class should not be used because they block the therapeutic action of dopamine.

*eliminate options 3 and 4 because they are comparable or alike*

  1. 4 g daily Rationale:

Aspirin may be used to treat the client with rheumatoid arthritis. It may also be used to reduce the risk of recurrent transient ischemic attack (TIA) or brain attack (stroke) or reduce the risk of myocardial infarction (MI) in clients with unstable angina or a history of a previous MI. The normal dose for clients being treated with aspirin to decrease thrombosis and MI is 300 to 325 mg/day. Clients being treated to prevent TIAs are usually prescribed 1.3 g/day in two to four divided doses. Clients with rheumatoid arthritis are treated with 3.6 to 5.4 g/day in divided doses. *Eliminate options 1 and 4 because they are alike*

  1. Complaints of a metallic taste in the mouth
  2. ) A health care provider prescribes auranofin (Ridaura) for a

Rationale: client with rheumatoid arthritis. Which of the following would

Ridaura is the one gold preparation that is given orally rather indicate to the nurse that the client is experiencing toxicity than by injection. Gastrointestinal reactions including related to the medication?

  1. Joint pain diarrhea, abdominal pain, nausea, and loss of appetite are
  2. Constipation common early in therapy, but these usually subside in the first
  3. Ringing in the ears 3 months of therapy. Early symptoms of toxicity include a
  4. Complaints of a metallic taste in the mouthrash, purple blotches, pruritus, mouth lesions, and a metallic taste in the mouth.

 

  1. "Swallow the tablets with large amounts of water or milk."
  1. ) A film-coated form of diflunisal has been prescribed for a

Rationale: client for the treatment of chronic rheumatoid arthritis. The client

Diflunisal may be given with water, milk, or meals. The tablets calls the clinic nurse because of difficulty swallowing the tablets. should not be crushed or broken open. Taking the medication Which initial instruction should the nurse provide to the client?

  1. "Crush the tablets and mix them with food." with a large amount of water or milk should be tried before
  2. "Notify the health care provider for a medication change." contacting the health care provider.
  3. "Open the tablet and mix the contents with food."  
  4. "Swallow the tablets with large amounts of water or milk."  
  5. 400 mg orally three times a day
  1. ) A health care provider instructs a client with rheumatoid

Rationale: arthritis to take ibuprofen (Motrin). The nurse reinforces the

For acute or chronic rheumatoid arthritis or osteoarthritis, the instructions, knowing that the normal adult dose for this client is normal oral adult dose is 400 to 800 mg three or four times which of the following?

  1. 100 mg orally twice a day
  2. 200 mg orally twice a day  
  3. 400 mg orally three  times a day       
  4. 1000 mg orally four times a day  

220. 220.) A adult client with muscle spasms is taking an oral maintenance dose of baclofen (Lioresal). The nurse reviews the medication record, expecting that which dose should be prescribed?

1.  15 mg four times a day

2.  25 mg four times a day

3.  30 mg four times a day              

4.  40 mg four times a day              

1. 15 mg four times a day Rationale:

Baclofen is dispensed in 10- and 20-mg tablets for oral use. Dosages are low initially and then gradually increased.Maintenance doses range from 15 to 20 mg administered three or four times a day.

221. 221.) A nurse is reviewing the health care provider's prescriptions for an adult client who has been admitted to the hospital following a back injury. Carisoprodol (Soma) is prescribed for the client to relieve the muscle spasms; the health care provider has prescribed 350 mg to be administered four times a day. When preparing to give this medication, the nurse determines that this dosage is:

1.  The normal adult dosage

2.  A lower than normal dosage

3.  A higher than normal dosage

4.  A dosage requiring further clarification

1.The normal adult dosage Rationale:

The normal adult dosage for carisoprodol is 350 mg orally three or four times daily.

  • ) A nurse has administered a dose of diazepam (Valium) to a client. The nurse would take which important action before leaving the client's room?
    1. Giving the client a bedpan
    2. Drawing the shades or blinds closed
    3. Turning down the volume on the television Per agency policy, putting up the side rails on the bed

 

  • ) A client with a psychotic disorder is being treated with haloperidol (Haldol). Which of the following would indicate the presence of a toxic effect of this medication?
    1. Nausea
    2. Hypotension
    3. Blurred vision
    4. Excessive salivation
  • ) Neuroleptic malignant syndrome is suspected in a client who is taking chlorpromazine. Which medication would the nurse prepare in anticipation of being prescribed to treat this adverse effect related to the use of chlorpromazine? 1. Protamine sulfate
    1. Bromocriptine (Parlodel)
    2. Phytonadione (vitamin K)
    3. Enalapril maleate (Vasotec)
  • ) A nursing student is assigned to care for a client with a diagnosis of schizophrenia. Haloperidol (Haldol) is prescribed for the client, and the nursing instructor asks the student to describe the action of the medication. Which statement by the nursing student indicates an understanding of the action of this medication? 1. It is a serotonin reuptake blocker.
    1. It inhibits the breakdown of released acetylcholine.
    2. It blocks the uptake of norepinephrine and serotonin.
    3. It blocks the binding of dopamine to the postsynaptic dopamine receptors in the brain.
    4. Per agency policy,puttingup the side rails on the bed Rationale:

Diazepam is a sedative-hypnotic with anticonvulsant and skeletal muscle relaxant properties. The nurse should institute safety measures before leaving the client's room to ensure that the client does not injure herself or himself. The most frequent side effects of this medication are dizziness, drowsiness, and lethargy. For this reason, the nurse puts the side rails up on the bed before leaving the room to prevent falls. Options 1, 2, and 3 may be helpful measures that provide a comfortable, restful environment, but option 4 is the one that provides for the client's safety needs.

  1. Excessive salivation Rationale:

Toxic effects include extrapyramidal symptoms (EPS) noted as marked drowsiness and lethargy, excessive salivation, and a fixed stare. Akathisia, acute dystonias, and tardive dyskinesia are also signs of toxicity. Hypotension, nausea, and blurred vision are occasional side effects.

  1. Bromocriptine (Parlodel) Rationale:

Bromocriptine is an antiparkinsonian prolactin inhibitor used in the treatment of neuroleptic malignant syndrome. Vitamin K is the antidote for warfarin (Coumadin) overdose. Protamine sulfate is the antidote for heparin overdose. Enalapril maleate is an antihypertensive used in the treatment of hypertension.

  1. It blocks the binding of dopamine to the postsynaptic dopamine receptors in the brain.

Rationale:

Haloperidol acts by blocking the binding of dopamine to the postsynaptic dopamine receptors in the brain. Imipramine hydrochloride (Tofranil) blocks the reuptake of norepinephrine and serotonin. Donepezil hydrochloride (Aricept) inhibits the breakdown of released acetylcholine. Fluoxetine hydrochloride (Prozac) is a potent serotonin reuptake blocker.

226. 226.) A client receiving lithium carbonate (Lithobid) complains of loose, watery stools and difficulty walking. The nurse would expect the serum lithium level to be which of the following? 1. 0.7 mEq/L

2.  1.0 mEq/L

3.  1.2 mEq/L

4.  1.7 mEq/L

4.1.7 mEq/L Rationale:

The therapeutic serum level of lithium ranges from 0.6 to 1.2 mEq/L. Serum lithium levels above the therapeutic level will produce signs of toxicity.

  1. Not occur for 2 to [19][20]weeks of administration
  1. ) When teaching a client who is being started on

Rationale: imipramine hydrochloride (Tofranil), the nurse would

The therapeutic effects of administration of imipramine hydrochloride may inform the client that the desired effects of the not occur for 2 to 3 weeks after the antidepressant therapy has been medication may:

  1. Start during the first week of administration Therefore options [21][22][23], [24], and [25] are incorrect.
  2. Not occur for 2 to 3 weeks of administration  
  3. Start during the second week of administration  
  4. Not occur until after a month of administration  
  5. Postural hypotension
  1. ) A client receiving an anxiolytic medication

Rationale: complains that he feels very "faint" when he tries to

Anxiolytic medications can cause postural hypotension. The client needs get out of bed in the morning. The nurse recognizes to be taught to rise to a sitting position and get out of bed slowly this complaint as a symptom of:

  1. Cardiac dysrhythmias because of this adverse effect related to the medication. Options 1, 3, and
  2. Postural hypotension 4 are unrelated to the use of this medication.
  3. Psychosomatic symptoms  
  4. Respiratory insufficiency  
  5. Instructing theclient to call for ambulation assistance
  1. ) A client is placed on chloral hydrate

Rationale:

(Somnote) for short-term treatment. Which

Chloral hydrate (a sedative-hypnotic) causes sedation and impairment of nursing action indicates an understanding of the motor coordination; therefore, safety measures need to be implemented. The major side effect of this medication?

  1. Monitoring neurological signs every 2 hours client is instructed to call for assistance with ambulation. Options 1 and 2 are
  2. Monitoring the blood pressure every 4 hours not specifically associated with the use of this medication. Although option 4
  3. Instructing the client to call for ambulation is an appropriate nursing intervention, it is most important to instruct the client to call for assistance with ambulation. assistance
  4. Lowering the bed and clearing a path to the bathroom at bedtime        
    1. Frequent handwashing with hot, soapy water
  5. ) A client admitted to the hospital gives the

Rationale: nurse a bottle of clomipramine (Anafranil). The

Clomipramine is commonly used in the treatment of obsessive-compulsive nurse notes that the medication has not been disorder. Handwashing is a common obsessive-compulsive behavior. Weight taken by the client in 2 months. What behaviors gain is a common side effect of this medication. Tachycardia and sedation are observed in the client would validate side effects. Insomnia may occur but is seldom a side effect. noncompliance with this medication?

  1. Complaints of hunger  
  2. Complaints of insomnia  
  3. A pulse rate less than 60 beats per minute  
  4. Frequent handwashing with hot, soapy water  
    1. The client's orientation and delusional status
  5. ) A client in the mental health unit is

Rationale: administered haloperidol (Haldol). The nurse

Haloperidol is used to treat clients exhibiting psychotic features. Therefore, to would check which of the following to determine determine medication effectiveness, the nurse would check the client's medication effectiveness?

  1. The client's vital signs orientation and delusional status. Vital signs are routine and not specific to this
  2. The client's nutritional intake The physical safety of other clients is not a direct assessment of this
  3. The physical safety of other unit clients Monitoring nutritional intake is not related to this situation.
  4. The client's orientation and delusional status
  5. Addictive properties are enhanced in the presence of psychotropic
  1. ) Diphenhydramine hydrochloride (Benadryl)

medications. is used in the treatment of allergic rhinitis for a

Rationale: hospitalized client with a chronic psychotic

The addictive properties of diphenhydramine hydrochloride are enhanced disorder. The client asks the nurse why the when used with psychotropic medications. Allergic symptoms may not be medication is being discontinued before hospital short term and will occur if allergens are present in the environment. Poor discharge. The nurse responds, knowing that:

  1. Allergic symptoms are short in duration. compliance may be a problem with psychotic clients but is not the subject of
  2. This medication promotes long-term the question. Diphenhydramine hydrochloride may be used for extrapyramidal symptoms and mild medication-induced movement disorders. extrapyramidal symptoms.
  3. Addictive properties are enhanced in the presence of psychotropic medications.  
  4. Poor compliance causes this medication to fail to reach its therapeutic blood level.  

234. 234.) A hospitalized client is started on phenelzine sulfate (Nardil) for the treatment of depression. At lunchtime, a tray is delivered to the client. Which food item on the tray will the nurse remove?

1.  Yogurt

2.  Crackers

3.  Tossed salad

4.  Oatmeal cookies

1. Yogurt

Rationale:

Phenelzine sulfate is a monoamine oxidase inhibitor (MAOI). The client should avoid taking in foods that are high in tyramine. These foods could trigger a potentially fatal hypertensive crisis. Foods to avoid include yogurt, aged cheeses, smoked or processed meats, red wines, and fruits such as avocados, raisins, or figs.

  1. ) A tricyclic antidepressant is administered to a client daily. The nurse plans to monitor for the common side effects of the medication and includes which of the following in the plan of care?
    1. Offer hard candy or gum periodically.
    2. Offer a nutritious snack between meals.
    3. Monitor the blood pressure every 2 hours.

  • Review the white blood cell (WBC) count results daily.
  1. ) A client is being treated for depression with amitriptyline hydrochloride. During the initial phases of treatment, the most important nursing intervention is:
    1. Prescribing the client a tyramine-free diet Checking the client for anticholinergic effects 3. Monitoring blood levels frequently because there is a narrow range between therapeutic and toxic blood levels of this medication 4. Getting baseline postural blood pressures before administering the medication and each time the medication is administered
  2. ) A client who is on lithium carbonate (Lithobid) will be discharged at the end of the week. In formulating a discharge teaching plan, the nurse will instruct the client that it is most important to:
    1. Avoid soy sauce, wine, and aged cheese.
    2. Have the lithium level checked every week.
    3. Take medication only as prescribed because it can become addicting.
    4. Check with the psychiatrist before using any over-the- counter (OTC) medications or prescription medications.
    5. Offer hard candy or gum periodically.

Rationale:

Dry mouth is a common side effect of tricyclic antidepressants. Frequent mouth rinsing with water, sucking on hard candy, and chewing gum will alleviate this common side effect. It is not necessary to monitor the blood pressure every 2 hours. In addition, it is not necessary to check the WBC daily. Weight gain is a common side effect and frequent snacks will aggravate this problem.

  1. Getting baseline postural blood pressures before administering the medication and each time the medication is administeredRationale:

Amitriptyline hydrochloride is a tricyclic antidepressant often used to treat depression. It causes orthostatic changes and can produce hypotension and tachycardia. This can be frightening to the client and dangerous because it can result in dizziness and client falls. The client must be instructed to move slowly from a lying to a sitting to a standing position to avoid injury if these effects are experienced. The client may also experience sedation, dry mouth, constipation, blurred vision, and other anticholinergic effects, but these are transient and will diminish with time.

4.Check with the psychiatrist before using any over-the-counter (OTC) medications or prescription medications.

Rationale:

Lithium is the medication of choice to treat manic-depressive illness. Many OTC medications interact with lithium, and the client is instructed to avoid OTC medications while taking lithium. Lithium is not addicting, and, although serum lithium levels need to be monitored, it is not necessary to check these levels every week. A tyramine-free diet is associated with monoamine oxidase inhibitors.

  1. Via facemask
  1. ) Ribavirin (Virazole) is prescribed for the

Rationale: hospitalized child with respiratory syncytial virus (RSV).

Ribavirin is an antiviral respiratory medication used mainly in

The nurse prepares to administer this medication via hospitalized children with severe RSV and in high-risk children. which of the following routes?

  1. Orally Administration is via hood, face mask, or oxygen tent. The medication
  2. Via face mask is most effective if administered within the first 3 days of the infection.
  3. Intravenously  
  4. Intramuscularly  
    1. Wearing goggles
  5. ) Which of the following precautions will the nurse

Rationale: specifically take during the administration of ribavirin

Some caregivers experience headaches, burning nasal passages and

(Virazole) to a child with respiratory syncytial virus eyes, and crystallization of soft contact lenses as a result of (RSV)?

  1. Wearing goggles administration of ribavirin. Specific to this medication is the use of Wearing a gown goggles. A gown is not necessary. A mask may be worn. Handwashing
  2. Wearing a gown and a mask is to be performed before and after any child contact.
  3. Handwashing before administration
    1. Dry mouth
  4. ) A client with Parkinson's disease has been

Rationale: prescribed benztropine (Cogentin). The nurse monitors

Common GI side effects of benztropine therapy include constipation for which gastrointestinal (GI) side effect of this and dry mouth. Other GI side effects include nausea and ileus. These medication?

  1. Diarrhea effects are the result of the anticholinergic properties of the
  2. Dry mouth
  3. Increased appetite *Eliminate options 1 and 4 because they are comparable or alike.
  4. Hyperactive bowel sounds Recall that the medication is an anticholinergic, which causes dry mouth*

 

  1. Leads to physical and psychological dependence with prolonged
  1. ) A client with a history of simple partial seizures is

high-dose therapy taking clorazepate (Tranxene), and asks the nurse if

Rationale: there is a risk of addiction. The nurse's response is based

Clorazepate is classified as an anticonvulsant, antianxiety agent, and on the understanding that clorazepate:

  1. Is not habit forming, either physically or sedative-hypnotic (benzodiazepine). One of the concerns with clorazepate therapy is that the medication can lead to physical or psychologically
  2. Leads to physical tolerance, but only after 10 or more psychological dependence with prolonged therapy at high doses. For this reason, the amount of medication that is readily available to the years of therapy
  3. Leads to physical and psychological dependence with client at any one time is restricted.

*Eliminate options 2 and 4 first because of the closed-ended word prolonged high-dose therapy

  1. Can result in psychological dependence only, "only"* because of the nature of the medication
    1. Are worse during initial therapy and decrease or disappear with
  2. ) A client who was started on anticonvulsant therapy long-term use

with clonazepam (Klonopin) tells the nurse of increasing

Rationale:

clumsiness and unsteadiness since starting the

Drowsiness, unsteadiness, and clumsiness are expected effects of the medication. The client is visibly upset by these medication during early therapy. They are dose related and usually manifestations and asks the nurse what to do. The nurse's diminish or disappear altogether with continued use of the medication.

response is based on the understanding that these

It does not indicate that a severe side effect is occurring. It is also symptoms:

  1. Usually occur if the client takes the medication with unrelated to interaction with another medication. The client is encouraged to take this medication with food to minimize food
  2. Are probably the result of an interaction with another gastrointestinal upset.

*Eliminate options 2 and 3 first because they are comparable or

medication

  1. Indicate that the client is experiencing a severe alike and because of the word "severe" in option 3* untoward reaction to the medication
  2. Are worse during initial therapy and decrease or disappear with long-term use
  1. ) A hospitalized client is having the dosage of clonazepam (Klonopin) adjusted. The nurse should plan to:
    1. Weigh the client daily.
    2. Observe for ecchymosis.
    3. Institute seizure precautions.
    4. Monitor blood glucose levels.
  2. ) A client has a prescription for valproic acid (Depakene) orally once daily. The nurse plans to:
    1. Administer the medication with an antacid.
    2. Administer the medication with a carbonated beverage.
    3. Ensure that the medication is administered at the same time each day.
    4. Ensure that the medication is administered 2 hours before breakfast only, when the client's stomach is empty.
  3. ) A client taking carbamazepine (Tegretol) asks the nurse what to do if he misses one dose. The nurse responds that the carbamazepine should be:
    1. Withheld until the next scheduled dose
    2. Withheld and the health care provider is notified immediately Taken as long as it is not immediately before the next dose 4. Withheld until the next scheduled dose, which should then be doubled
    3. Institute seizure precautions.

Rationale:

Clonazepam is a benzodiazepine used as an anticonvulsant. During initial therapy and during periods of dosage adjustment, the nurse should initiate seizure precautions for the client. Options 1, 2, and 4 are not associated with the use of this medication.  

  1. Ensure that the medication is administered at the same time each day. Rationale:

Valproic acid is an anticonvulsant, antimanic, and antimigraine medication. It may be administered with or without food. It should not be taken with an antacid or carbonated beverage because these products will affect medication absorption. The medication is administered at the same time each day to maintain therapeutic serum levels.

*Use general pharmacology guidelines to assist in eliminating options 1 and 2. Eliminate option 4 because of the closed-ended word "only."*

  1. Taken as long as it is not immediately before the next dose Rationale:

Carbamazepine is an anticonvulsant that should be taken around the clock, precisely as directed. If a dose is omitted, the client should take the dose as soon as it is remembered, as long as it is not immediately before the next dose. The medication should not be double dosed. If more than one dose is omitted, the client should call the health care provider.

246. -afil

Erectile dysfunctions/s: headache, heartburn, diarrhea, flushing, nosebleeds, parathesias, changes in color vision

Contradicted in clients taking nitrates, anticoags, anti HTN

Common meds- sildenafil (viagra)

247.  albuterol

Bronchodilator

S/S: tachcardia, palpitations, tremors

248. alteplase (activase, tPA)

dissolves clots

TX: acute MI, DVT, massive PE, ischemic stroke

S/S: serious bleeding risks from recent wounds, puncture sites, weakened vessels, hypotension

NI: Must take 4-6 hrs of onset

249. amitripytyline (elavil)

TCA

S/S: anticholenergic effects, sedation, toxicity

NI: DO NOT admin with MAOIs, avoid alcohol, contradicted in clients w/ seizures

  1. -arin Anticoagulant

inhibit clotting factors (warfarin = factors VII, IX, X)

TX: evolving stroke, pulmonary embolism, massive deep vein thrombosis, cardiac cath, MI, DIC

S/S: hemorrhage, heparin induced thrombocytopenia, toxicity/overdose Common meds- warfarin (coumadin) {admin once daily, avoid NSAIDs & aspirin}, enoxaparin (lovenox)

251. -ase

Thrombolyticdissolves clots

TX: acute MI, DVT, massive PE, ischemic stroke

S/S: serious bleeding risks from recent wounds, puncture sites, weakened vessels, hypotension

NI: Must take 4-6 hrs of onset

Common meds- alteplase (activase, tPA)

252. -asone, -solone

- onide

Pred-

Cort-

Corticosteroid

prevent inflammatory response

S/S: Hyperglycemia, peptic ulcer, fluid retention (increased appetite), withdrawal symptoms, euphoria, insomnia, psychotic behavior

NI: admin w/ meals, DO NOT take with NSAIDS, teach DO NOT stop abruptly Common meds- prednisone (deltasone), betamethasone (celestone), hydrocortisone sodium succinate (Solu-cortef), Methylprednisolone sodium succinate (solu-medrol), fluticasone propionate (advair, flovent)

253. -azine

- setron

Antiemtic reduce N & V

S/S: drowsiness, anticholenergic effects, restlessness, tardive dyskinesia, EPS NI: monitor VS

Common meds- promethazine (phenergan), metaoclopramide (reglan), ondansertron

(zofran)

254. (biguanide) Metformin

Oral hypoglycemic

Used in conjunction with diet & exercise; type II NI: teach s/s of hypoglycemia, HbA1Cmetformin (glucophage): withhold 48 hrs before/after test w/ contrast

255. catopril, lisinopril, enalapril (vastotec)

Block the conversion of angiotensin I to angiotensin II

TX: HTN, HF, MI, diabetic nephropathy

S/S: Anigoedema, Cough, Electrolyte imbalance (^k+)

NI: Monitor K+ levels, BP

256. -cillin

Penicillin

TX: pneumonia, upper respiratory infections, septicemia, endocarditis, rheumatic fever, GYN infections

NI: hypersensitivity w/ poss. anaphylaxis

257. -cycline,

-floxacin

Antibiotic

258. Digoxin side effects

-Fatigue

-Bradycardia

-Anorexia

-Nausea/Vomiting

259. diphenhydramine (benadryl), loratadine

(claratin), cetirizine (zyrtec), fexofenadrine (allegra)

S/S: anticholenergic effects (cant see, spit, pee, poop), drowsiness

NI: use cautiously pts w/ HTN, PUD, urinary retention, assess hypokalemia, BP, Advise to take @ night

  1. -dipine Ca+ channel blocker

Slows movement of calcium into smooth muscle= arterial dilation & decreased BP

Tx: angina, HTN (verapamil & diltiazem may be used for AFIB, A flutter, SVT

S/S: Constipation, reflex tachycardia, peripheral edema, toxicity

Common meds- nifedipine (procardia), verapamil, diltiazem

261. Duloxetine (Cymbalta) Fluoxetine (Provac)

Escitalopram (Lexapro)

Sertraline (Zoloft)

S/S: weight gain, fatigue, sexual dysfunction, drowsiness

NI: avoid alcohol, do not discontinue abrubptly, monitor for serotonin syndrome! (agitation, confusion, hallucinations) within first 72 hrs

262. Food to avoid when taking Lithium

-salty foods

-alcoholic beverages

263. Fosomax

same as-Alendronate is used for treating osteoporosis in men and postmenopausal women.

264. Garamycin-

Antibiotic that is toxic to the kidney, injected for radiology studies.

265. gentamicin sulfate

TX: pneumonia, meningitis, septicemia

NI: high risk for ototoxicity, nephrotoxicity, monitor creatinine & BUN

266. -gliptin

       -glitazone                                             

Diabetes Mellitus

267. Haldol-inform if you are taking ____________ medication.

-benzodiazepine class of anti-anxiety drugs (all ending with "pam") and even, Xanax.

268. HPV vaccine

Human Papilloma Virus (HPV2, HPV4) - -Three doses should be given over a 6 month-interval for females at 11 to 12 years of age (minimum age is 9 years).

-The second dose should be administered 2 months after the first dose, and the third dose should be administered 6 months after the first dose.

-HPV4 may be given to males starting at age 9 years of age.

269. -ide

Oral hypoglycemic

Used in conjunction with diet & exercise; type II NI: teach s/s of hypoglycemia, HbA1Cmetformin (glucophage): withhold 48 hrs before/after test w/ contrast

270. -iprazole

-apine

-idone

Second Generation Antipsychotic (SGA)

  1. Know about Transdermal patch -• Apply at the same time once each day,

preferably in the morning. Keep patch on for 12 to 14 hr each day.

  • Remove the patch at night to reduce the risk of developing tolerance to nitroglycerin. Be medication-free a minimum of 10 to 12 hr each day (usually at night).
  • Do not cut patches to ensure appropriate dosage.
  • Place the patch on a hairless area of skin (chest, back, or abdomen) and rotate sites to prevent skin irritation.
  • Wash skin with soap and water and dry thoroughly before applying new patch.

272. Labs for patients taking hydrothiazide

Periodic determination of serum electrolytes to detect possible electrolyte imbalance should be done at appropriate intervals.

273. Lipitor

-lowers cholesterol in blood, "statins". Reduce LDL and total cholesterol. Raise HDL.

  1. Lisinopril therapeutic effect blood pressure answer (e.g. 120/80)

275.  Lithium report immediately

slurred speech

276.  lorazepam

TX: Sedative-hypnotics for sleep, Adjuncts to anesthesia to induce relaxation and amnesia (procedural memory loss), To reduce anxiety (anxiolytic), Panic disorders, To treat or prevent seizures, For alcohol withdrawal, Muscle relaxant

277. lovastatin (mevacor)

aid in lowering LDL & increasing HDL

S/S: muscle aches, hepatotoxicity, myopathy, rhabdomyolysis, peripheral neruopathy

NI: take in evening, monitor renal and liver function, low fat/high fiber diet, drug interactions: digoxin, warfarin, NSAIDs, etc.

278. Macrodantin medication

used to treat or prevent certain urinary tract infections

279. Medication for

       Schizophrenia                            

risperidone, Risperdal

280. metropolol, labetalol, propanolol

inhibit stimulation of receptor sites= decreased cardiac excitability, CO, myocaridal O2 demand, lower BP by decreasing release of renin in the kidney

TX: HTN, angina, tachydysryhmias, HF, MI

S/S: Bradycardia, Bradypena, Bronchospasms, decreased BP

NI: Monitor DM for hypoglycemia

281. -mycin

Aminoglycoside

(Antimicrobials)

TX: pneumonia, meningitis, septicemia

NI: high risk for ototoxicity, nephrotoxicity, monitor creatinine & BUN 

Common meds- gentamicin sulfate (garamycin) therapeutic range: 4-12mcg/dL

282. NEUPOGEN (filgrastim)- what is the appropriate  route of this med?  

administered by subcutaneous injection or IV infusion

283. nifedipine (procardia), verapamil, diltiazem

Slows movement of calcium into smooth muscle= arterial dilation & decreased BP

Tx: angina, HTN (verapamil & diltiazem may be used for AFIB, A flutter, SVT

S/S: Constipation, reflex tachycardia, peripheral edema, toxicity

284. -olol

Beta Blockerinhibit stimulation of receptor sites= decreased cardiac excitability, CO, myocaridal O2 demand, lower BP by decreasing release of renin in the kidney

TX: HTN, angina, tachydysryhmias, HF, MI

S/S: Bradycardia, Bradypena, Bronchospasms, decreased BP

NI: Monitor DM for hypoglycemia

Common meds- metropolol, labetalol, propanolol

285. omepazole (prilosec)

S/S: D,V, N, can increase risk for fractures,, pneumonia, & acid reboundNI: DO NOT crush, chew, break, notify PROVIDER if GI bleeding!

286. Opioid toxicity-what to check first

oxygen saturation

287. -pam, -lam

Benzodiazipines

TX: Sedative-hypnotics for sleep, Adjuncts to anesthesia to induce relaxation and amnesia (procedural memory loss), To reduce anxiety (anxiolytic), Panic disorders, To treat or prevent seizures, For alcohol withdrawal, Muscle relaxant

288. Patient identifiers

-Medical record number

-home telephone number

289. Patient reports IV discomfort, what is your first action?

color and temperature

291. -phylline,

-terol

Bronchodilator

S/S: tachcardia, palpitations, tremors

Common meds- albeuterol

292. -pram, -ine

SSRIs

S/S: weight gain, fatigue, sexual dysfunction, drowsiness

NI: avoid alcohol, do not discontinue abrubptly, monitor for serotonin syndrome! (agitation, confusion, hallucination within first 72 hrs

293. -prazole

Proton pump inhibitor

S/S: D,V, N, can increase risk for fractures,, pneumonia, & acid rebound NI: DO NOT crush, chew, break, notify PROVIDER if GI bleeding!

Common meds- omepazole (prilosec)

294. Prednisone report

sore throat

295. -pril

ACE inhibitor

Block the conversion of angiotensin I to angiotensin II

TX: HTN, HF, MI, diabetic nephropathy

S/S: Anigoedema, Cough, Electrolyte imbalance (^k+)

NI: Monitor K+ levels, BP

Common med- catopril, lisinopril, enalapril (vastotec)

296. promethazine (phenergan), metaoclopramide

(reglan),          ondansertron                 

(zofran)

reduce N & V

S/S: drowsiness, anticholenergic effects, restlessness, tardive dyskinesia, EPS

NI: monitor VS

297. RBC Blood transfusion

http://www.atitesting.com/ati_next_gen/FocusedReview/data/datacontext/RM%20AMS%20RN%208.0%20Chp%2044.p (prime with normal saline and infuse with sodium chloride).

298. rednisone (deltasone), betamethasone

(celestone), hydrocortisone sodium succinate

(Solu-cortef),

Methylprednisolone sodium succinate (solu-medrol), fluticasone propionate (advair, flovent)

prevent inflammatory response

S/S: Hyperglycemia, peptic ulcer, fluid retention (increased appetite), withdrawal symptoms, euphoria, insomnia, psychotic behavior

NI: admin w/ meals, DO NOT take with NSAIDS, teach DO NOT stop abruptly

299.  sildenafil (viagra)

s/s: headache, heartburn, diarrhea, flushing, nosebleeds, parathesias, changes in color vision

Contradicted in clients taking nitrates, anticoags, anti HTN

300. Singulair

used before exercise to prevent breathing problems during exercise (bronchospasm).

301. -statin

Antilipidemic

aid in lowering LDL & increasing HDL

S/S: muscle aches, hepatotoxicity, myopathy, rhabdomyolysis, peripheral neruopathy

NI: take in evening, monitor renal and liver function, low fat/high fiber diet, drug interactions: digoxin, warfarin, NSAIDs, etc.

Common meds- lovastatin (mevacor)

  1. Penicillin TX: pneumonia, upper respiratory infections, septicemia, endocarditis, rheumatic fever, GYN infections

NI: hypersensitivity w/ poss. anaphylaxis

Sumatriptan (treats migraine headaches) adverse effect pain, tightness, pressure, or heaviness in the chest, throat, neck, and/or jaw

 slow or difficult speech

303. -tidine

Antiulcer

S/S: lethargy, depression, confusion, decreased libido

Common meds- ranitidine hydrochloride (zantac), cimetidine (tagamet), famotidine (pepcid)

304. -tyline

Tricyclic antidepressant

S/S: anticholenergic effects, sedation, toxicity

NI: DO NOT admin with MAOIs, avoid alcohol, contradicted in clients w/ seizures

Common meds- amitripytyline (elavil)

305. Valporic Acid lab

liver

306. -vir

Antiviral

307. warfarin (coumadin) {admin once daily, avoid NSAIDs & aspirin}, enoxaparin (lovenox)

inhibit clotting factors (warfarin = factors VII, IX, X)

TX: evolving stroke, pulmonary embolism, massive deep vein thrombosis, cardiac cath, MI, DIC

S/S: hemorrhage, heparin induced thrombocytopenia, toxicity/overdose

308.  What food should you increase when taking Lasix?

-increased amounts of potassium-rich foods (e.g., bananas, prunes,raisins, and orange juice)

309. What lab values should a nurse monitor for a patient with chronic renal failure?

■ Urinalysis

☐Hematuria, proteinuria, and alterations in specific gravity

☐Serum creatinine

-              Gradual increase of 1 to 2 mg/dL per every 24 to 48 hr for acute renalfailure (ARF)

-              Gradual increase over months to years for chronic renal failure (CRF) exceeding 4 mg/dL

■ Blood urea nitrogen (BUN)

-              80 to 100 mg/dL within 1 week with ARF

-              Gradual increase with elevated serum creatinine over months to years for

CRF

-              180-200 mg/dL with (CRF)

■ Serum electrolytes

-              Decreased sodium (dilutional) and calcium, increased potassium, phosphorus, and magnesium ■ Complete blood count (CBC)

-              Decreased hemoglobin

310.  What medication to administer with Tylenol overdose?

acetylcysteine (Mucomyst) must be given IV

311.   What to understand about Parkinson's Meds?

-they don't cure disease, they slow the process.

312. -zine

Antihistamine

S/S: anticholenergic effects (cant see, spit, pee, poop), drowsiness NI: use cautiously pts w/ HTN, PUD, urinary retention, assess hypokalemia, BP, Advise to take @ night

Common meds- diphenhydramine (benadryl), loratadine (claratin), cetirizine (zyrtec), fexofenadrine (allegra)

313. -zosin

HTN/Prostate

Powered by TCPDF (www.tcpdf.org)

[1] . Prednisone

[2] . 30.) A client with diabetes mellitus visits a health

Rationale: care clinic. The client's diabetes mellitus previously

Prednisone may decrease the effect of oral hypoglycemics, insulin, had been well controlled with glyburide (DiaBeta) diuretics, and potassium supplements. Option 2, a monoamine oxidase daily, but recently the fasting blood glucose  level inhibitor, and option 3, a β-blocker, have their own intrinsic hypoglycemic has been 180 to 200 mg/dL. Which medication, if activity. Option 4 decreases urinary excretion of sulfonylurea agents, added to the client's regimen, may have contributed causing increased levels of the oral agents, which can lead to to the hyperglycemia?

[3] 1. Prednisone                                                                              hypoglycemia.

[4] . Phenelzine (Nardil)

[5] . Atenolol (Tenormin)

[6] . Allopurinol (Zyloprim)

[7] .Diarrhea can occur secondary to the metformin.

[8] .The repaglinide is not taken if a meal is skipped.

[9] .The repaglinide is taken 30 minutes before eating.

[10] .Candy or another simple sugar is carried and used to treat mild hypoglycemia episodes.

Rationale:

Repaglinide is a rapid-acting oral hypoglycemic agent that stimulates pancreatic insulin secretion that should be taken before meals, and that should be withheld if the client does not eat. Hypoglycemia is a side effect of repaglinide and the client should always be prepared by carrying a simple sugar with her or him at all times. Metformin is an oral hypoglycemic given in combination with repaglinide and works by decreasing hepatic glucose production. A common side effect of metformin is diarrhea. Muscle pain may occur as an adverse effect from metformin but it might signify a more serious condition that warrants health care provider notification, not the use of acetaminophen.

[11] .The potential for decreased effectiveness of the birth control pills exists while taking phenytoin (Dilantin).

Rationale:

Phenytoin (Dilantin) enhances the rate of estrogen metabolism, which can decrease the effectiveness of some birth control pills. Options 1, 2, are 4 are not accurate.

[12] . Blood pressure and heart rate

[13] . 123.) A nurse is planning to administer

Rationale: amlodipine (Norvasc) to a client. The nurse

Amlodipine is a calcium channel blocker. This medication decreases the rate and force plans to check which of the following of cardiac contraction. Before administering a calcium channel blocking agent, the before giving the medication?

[14] 1. Respiratory rate          nurse should check the blood pressure and heart rate, which could both decrease in

[15] 2. Blood pressure and heart rate    response to the action of this medication. This action will help to prevent or identify

[16] 3. Heart rate and respiratory rateearly problems related to decreased cardiac contractility, heart rate, and conduction.

[17] . Level of consciousness and blood *amlodipine is a calcium channel blocker, and this group of medications decreases the rate and force of cardiac contraction. This in turn lowers the pulse rate and pressure blood pressure.*  

124. 124.) A client with chronic renal failure is receiving ferrous sulfate (Feosol). The nurse monitors the client for which common side effect associated with this medication?

1. Diarrhea

2. Weakness

3. Headache

4. Constipation

4.Constipation Rationale:

Feosol is an iron supplement used to treat anemia. Constipation is a frequent and uncomfortable side effect associated with the administration of oral iron supplements.

Stool softeners are often prescribed to prevent constipation.

*Focus on the name of the medication. Recalling that oral iron can cause constipation will easily direct you to the correct option.*

[18] .Scallops Rationale:

Colchicine is a medication used for clients with gout to inhibit the reabsorption of uric acid by the kidney and promote excretion of uric acid in the urine. Uric acid is produced when purine is catabolized. Clients are instructed to modify their diet and limit excessive purine intake. High-purine foods to avoid or limit include organ meats, roe, sardines, scallops, anchovies, broth, mincemeat, herring, shrimp, mackerel, gravy, and yeast.

[19] . The medication will be discontinued 1 to 2 days before the surgery and

[20] . 229.) A client who is taking lithium carbonate resumed as soon as full oral intake is allowed.

(Lithobid) is scheduled for surgery. The nurse informs

Rationale: the client that:

[21] 1. The medication will be discontinued a week before          The client who is on lithium carbonate must be off the medication for 1 to

[22] days before a scheduled surgical procedure and can resume the the surgery and resumed 1 week postoperatively.

[23] 2. The medication is to be taken until the day of       medication when full oral intake is prescribed after the surgery.

*lithium carbonate is an oral medication and is not given as an surgery and resumed by injection immediately injection* postoperatively.

[24] . The medication will be discontinued 1 to 2 days before the surgery and resumed as soon as full oral intake is allowed.

[25] . The medication will be discontinued several days before surgery and resumed by injection in the immediate postoperative period.

Buy ATI Pharmacology Proctored Test Answers Online

Talk to our expert to get the help with ATI Pharmacology Proctored Test to complete your assessment on time and boost your grades now

The main aim/motive of the management assignment help services is to get connect with a greater number of students, and effectively help, and support them in getting completing their assignments the students also get find this a wonderful opportunity where they could effectively learn more about their topics, as the experts also have the best team members with them in which all the members effectively support each other to get complete their diploma assignments. They complete the assessments of the students in an appropriate manner and deliver them back to the students before the due date of the assignment so that the students could timely submit this, and can score higher marks. The experts of the assignment help services at urgenthomework.com are so much skilled, capable, talented, and experienced in their field of programming homework help writing assignments, so, for this, they can effectively write the best economics assignment help services.

Get Online Support for ATI Pharmacology Proctored Test Assignment Help Online


Resources

  • 24 x 7 Availability.
  • Trained and Certified Experts.
  • Deadline Guaranteed.
  • Plagiarism Free.
  • Privacy Guaranteed.
  • Free download.
  • Online help for all project.
  • Homework Help Services
Copyright © 2009-2023 UrgentHomework.com, All right reserved.